WISE-MD

Lakukan tugas rumah & ujian kamu dengan baik sekarang menggunakan Quizwiz!

steps of pathophysiology of appendicitis

1. Obstruction of the lumen by lymphoid hyperplasia or fecalith 2. Continued mucous production with distention of the appendix 3. Venous obstruction 4. Arterial obstruction 5. Ischemia 6. Gangrene 7. Perforation

A 23 year old man presents with an 18 hour history of abdominal pain, now localized to the right lower quadrant. He complains of associated nausea and anorexia. What should be ordered?

IV LR, IV abx The working diagnosis in this patient is appendicitis. Intravenous hydration and antibiotics should be initiated promptly. Mechanical bowel prep, nasogastric tube decompression, and the need for blood products are not indicated in uncomplicated appendicitis.

CT scan demonstrating evidence of a walled-off appendiceal abscess that is 5cm in maximal diameter should be treated with: a) open appy b) admission for hydration and stabilization c) broad spectrum abx and interval appendectomy d) percutaneous drainage of abscess e) evaluation for a tumor obstructing the appendix

The CT scan demonstrates evidence of a walled-off appendiceal abscess that is 5 cm in maximal diameter. Percutaneous drainage of a peri-appendiceal abscess that is readily accessible and larger than 3 cm reduces the need of more extensive surgery and allows the inflammatory process to resolve, allowing a safer interval appendectomy in 4-6 weeks.

One day after undergoing laparoscopic gastric bypass, a 29-year-old woman is evaluated for severe tachycardia and left shoulder pain. The procedure was uncomplicated. The patient has a 10-year history of anxiety. On physical examination, temperature is 100.5 °F (38°C), pulse is 130/min, respirations are 28/min, and blood pressure is 106/74 mm Hg. The surgical site appears unremarkable. Which of the following is the most likely etiology? a) Anastomotic leak b) Marginal ulcer c) Bowel obstruction d) Panic attack e) Positional injury of left shoulder

a) Anastomotic leak In an obese patient peritoneal signs may not be evident. The left shoulder pain could be from diaphragmatic irritation and when combined with tachycardia mandates evaluation for a leak in a patient who has recently had a gastric bypass.

75 year old female with 4 weeks of intermittent post prandial right upper quadrant abdominal pain was seen by her primary care physician who ordered a CT scan and sent her home. He calls you urgently because the CT scan was read as "cholelithiasis with dilated appendix suspicious for acute appendicitis." The next step in management should be: a) Call the patient and schedule her for an elective colonoscopy b) Call the patient and schedule her for urgent appendectomy c) Call the patient and schedule her for urgent cholecystectomy d) Call the patient and tell her to come to the emergency room e) call the patient and ask to her to proceed to the lab for IBD testing

a) Call the patient and schedule her for an elective colonoscopy This patient does not have a history consistent with appendicitis - her history is that of biliary colic - which is confirmed with the CT scan findings of cholelithiasis. Although the appendix is abnormal by CT scan, since the clinical picture is not consistent with appendicitis, she should not have urgent appendectomy. The patient is not acutely ill so she does not need to go to the emergency room. Her biliary colic will be treated with elective cholecystectomy after the incidental findings of dilated appendix are worked up. Non-painful enlargement of the appendix in the elderly should raise the suspicion of colonic malignancy and a colonoscopy should be performed to rule this out. If this is negative, a discussion should be had with the patient about the possibilities of a appendiceal cystadenoma or cystadenocarcinoma and plans for elective resection made if she is an acceptable surgical candidate. Her gallbladder may be removed at the same time.

A 40-year-old woman who underwent a retrocolic laparoscopic gastric bypass 12 months ago presents to the emergency department with a 16-hour history of intermittent cramping pain in the mid abdomen that is exacerbated by meals. She also has nausea and vomiting, and she is now unable to tolerate liquids. At her most recent physical examination 3 weeks ago, the patient weighed 126 lb (57 kg), a 115-lb (52-kg) decrease from baseline before surgery. On physical examination in the emergency department, temperature is 98.6°F (37.0°C), pulse is 90/min, respirations are 16/min, and blood pressure is 116/76 mm Hg. The mucous membranes are somewhat dry. The mid abdomen is tender to palpation; there is no rebound tenderness or guarding. Results of laboratory studies are all normal. Computed tomography of the abdomen and pelvis shows dilated loops of small intestine and a dilated excluded stomach. Intravenous fluids are initiated. The most likely diagnosis of this problem is which of the following? a) Internal Hernia through transverse mesocolon b) Diaphragmatic hernia c) Pylorus stenosis d) Stricture of gastojeunostomy e) Foramen of Winslow hernia

a) Internal Hernia through transverse mesocolon

What structure does the left adrenal vein empty into? a) Left renal vein b) Inferior vena cava c) Right renal vein d) Left common iliac vein e) Left phrenic vein

a) Left renal vein The shorter distance between the right adrenal gland and inferior vena cava allows for a direct communication between the right adrenal vein and the inferior vena cava. This shorter distance can make control of the right adrenal vein technically challenging. The left adrenal vein, however, is longer and emptied directly into the left renal vein. Both adrenal glands have arterial blood supply via direct branches from the aorta, renal arteries, and phrenic arteries.

A 48 year-old male patient with refractory hypertension on four anti-hypertensive medications is found to have Cushing's syndrome. Review of his medications reveals no exogenous glucocorticoid administration. Which lab result would confirm that this patient is a candidate for laparoscopic adrenalectomy? a) Low ACTH b) Mild increase of cortisol in response to administration of corticotrophin releasing hormone c) High Cortisol d) Suppression of cortisol with low-dose dexamethasone e) High ACTH

a) Low ACTH Work-up of the patient with Cushing's syndrome requires identification of the source of the excess cortisol production. One would expect Cushing's syndrome of any etiology to have an elevated cortisol level. In a normal patient, cortisol levels would be suppressed by administration of low-dose dexamethasone. A high ACTH is indicative of a pituitary source, also known as Cushing's disease. A low ACTH level is indicative of a non-pituitary source; in this case making an adrenal source the most likely etiology.

A 19 year old man presents to the emergency department with progressive right lower quadrant abdominal pain over the last 24 hours, beginning while helping a friend move heavy furniture. He reports nausea, anorexia and fever to 101.2F at home. He has not had a bowel movement since the symptoms began. Which of the following components of the physical exam will be most useful in establishing the diagnosis: a) Palpation of the inguinal canal through the scrotum b) Asking the patient to sharply inhale while palpating under the liver edge c) Inspection of the abdomen d) Digital rectal examination e) Auscultation of the abdomen

a) Palpation of the inguinal canal through the scrotum It will be important to evaluate the patient for an incarcerated and strangulated inguinal hernia as an etiology for his fever and right lower quadrant abdominal pain, which can most effectively be performed by palpation of the inguinal canal through the scrotum. Observation may also demonstrate a bulge or erythematous skin changes in this region. Auscultation of the abdomen is unlikely to provide more information than direct palpation. Asking the patient to inhale sharply while palpating under the inferior edge of the liver to elicit Murphy's sign would be more important if the patient complained of right upper quadrant abdominal pain, or had symptoms of cholecystitis. Digital rectal examination may be helpful to evaluate for frank or occult blood, or tenderness, but is unlikely to provide more useful information than evaluation of the inguinal canal in this patient

A 37-year-old woman underwent an uneventful laparoscopic adjustable gastric band operation 6 months ago. She now presents to the emergency department with sudden onset of severe epigastric and retrosternal chest pain. She is vomiting and unable to tolerate any liquids by mouth. With the exception of a daily multivitamin, she takes no medications. On physical examination, her temperature is 98.4°F (36.9°C), pulse is 90/min and regular, respirations are 18/min, and blood pressure is 110/80 mm Hg. The lungs are clear to auscultation. Abdominal examination produces discomfort to epigastric palpation but no rebound, guarding, or organomegaly. Which of the following is the most likely cause of her symptoms? a) Slipped band b) Band erosion c) Port infection d) Psychological intolerance of band e) Pseudo-achalasia

a) Slipped band These symptoms are consistent with a slipped band in which the stomach herniates through the band and distension of the stomach causes gastric outlet obstruction. This is an emergency and if not promptly addressed can lead to ischemia, necrosis and perforation of the stomach

A 29 year old female presents to the emergency department during her 25th week of pregnancy with new onset of focal right lower quadrant pain. The pain has been constant for 8 hours. She vomited earlier and has not eaten today. A temperature of 37.5C is recorded. WBC is 16k. Ultrasound shows free fluid in the right lower quadrant, and an appendix dilated to 15mm. This exam also shows a normal, viable, intra-uterine pregnancy. Which of the following statements is true regarding her diagnosis and treatment options? a) The leukocytosis in this patient should be considered normal. b) This is the second most common surgical emergency in pregnancy. c) Regardless of treatment, there is at least a 25% rate of early delivery. d) CT scan of the abdomen/pelvis is necessary for definitive diagnosis. e) Diagnostic laparoscopy is not an option due to pregnancy

a) The leukocytosis in this patient should be considered normal. Acute appendicitis is the most common surgical emergency in pregnancy. Fetal loss after appendectomy is 4%. Early delivery after appendectomy is 7-10%. Up to 25% of appendectomies in pregnancy reveal a normal appendix. The highest negative appendectomy rate occurs in the second trimester; lowest in third. Physiologic leukocytosis in normal pregnancy is up to 16k. Non-ionizing radiation techniques typically preferred during pregnancy (ultrasound, MRI)

The patient is advised to have laparoscopic appendectomy. Which of the following statements regarding laparoscopic appendectomy is correct? a) Wound infection is a potential complication b) It is likely to require a 3-day hospitalization c) Patients should expect the placement of a drain after surgery d) The patient should not expect to return to work any sooner than after an open appendectomy e) It is not safe in early pregnancy

a) Wound infection is a potential complication Wound infection, though uncommon, is a known risk of laparoscopic appendectomy. It becomes more likely as the appendicitis becomes more advanced and complicated with extensive intra-abdominal infection. Drains are not used routinely following laparoscopic appendectomy. Hospitalization should generally be less than 36 hours. Pregnancy is not a contraindication to laparoscopic appendectomy. The patient should expect to return to normal activities sooner after a laparoscopic procedure.

A 40-year-old woman comes to the emergency department because of a 2-day history of severe periumbilical, colicky pain and abdominal distention. She is nauseated and has vomited light green vomitus 6 times in the past 24 hours. She has not passed stools or flatus for 48 hours. Her medical history is remarkable only for abdominal hysterectomy 8 years ago. On physical examination, her temperature is 100.4°F (38°C), pulse is 110/min and regular, respirations are 24/min, and blood pressure is 120/90 mm Hg. No murmurs are heard. The lungs are clear to auscultation and percussion. The abdomen is distended and mildly tender to deep palpation. Bowel sounds are high pitched and tinkling. Which of the following is the most likely diagnosis? a) adhesive small bowel obstruction b) Crohn's c) Diverticulitis d) Colon CA e) viral enteritis

a) adhesive small bowel obstruction Even at the young age of 40, the patient's surgical history puts her at risk for adhesive small bowel disease. This would be the most frequent cause of bowel obstruction. While crohn's is a possibility, it is less likely (especially in light of surgical history). Colon cancer, diverticulitis, and viral enteritis would typically not present with vomiting, distention, and the current listed abdominal exam.

A 45-year-old woman comes to the emergency department because of a 2-day history of severe periumbilical, colicky pain and abdominal distention. She is nauseated and has vomited light green vomitus 6 times in the past 24 hours. She has not passed stools or flatus for 48 hours. Her medical history is remarkable only for appendectomy at the age of 14 years. On physical examination, her temperature is 100.4°F (38.0°C), pulse is 100/min and regular, respirations are 24/min, and blood pressure is 120/90 mm Hg. No murmurs are heard. The lungs are clear to auscultation and percussion. The abdomen is distended and mildly tender with rebound on deep palpation. Bowel sounds are rare and high pitched. Laboratory studies show a leukocyte count of 15,000/mm3. Plain films of the abdomen show dilated proximal small bowel with pneumatosis. A nasogastric tube is placed and intravenous fluids are begun. Which of the following is the best next step in management? a) ex lap b) CT abd c) upper GI series with water soluble contrast d) barium enema e) observation

a) ex lap The presence of peritoneal signs and pneumatosis on x-rays are concerning for compromised or dead bowel. Especially in light of the leukocytosis and low grad temperature. In this setting, further workup would not alleviate this concern and thus exploratory laparotomy is the correct next step.

A 44-year-old woman who underwent laparoscopic cholecystectomy 36 hours ago comes to the emergency department because of a 8-hour history of severe periumbilical pain. She is nauseated and has vomited 4 times in the past 12 hours. On physical examination, the trochar site appears clean. Her temperature is 101.3°F (38.5°C), pulse is 118/min and regular, respirations are 22/min, and blood pressure is 120/90 mm Hg. No murmurs are heard. Diffuse crackles are heard at the bases of both lungs. The abdomen is distended and diffusely tender to palpation and percussion. Laboratory studies show a serum bilirubin of 0.8 mg/dL. Hepatobiliary iminodiacetic acid (HIDA) scan shows no evidence of a bile leak. Which of the following is the best next step in management? a) ex lap b) CT abd c) upper GI series with water soluble contrast d) abd u/s e) observation and ab

a) ex lap This patient is presenting with a picture of systemic sepsis and an acute abdomen. In the context of a recent surgery, abdomen cause of sepsis must lead the differential (in this case, mostly likely presenting as a result of a missed small bowel injury). Further work up, such as CT or UGI, would only delay the patient treatment and would not change the need for exploratory laparotomy.

A 78 year old woman presents to the Emergency Department with a 24 hour history of abdominal pain which has now localized to the right lower quadrant. On physical examination, there is tenderness of deep palpation in the right lower quadrant, without guarding or rebound. There is a palpable mass present in the right lower quadrant. Vital signs include a blood pressure of 135/83 mmHg, a pulse of 100 per minute, respirations of 18 per minute, and a temperature of 38.1° C. Which of the following could be included in the differential diagnosis? a) periappendiceal abscess b) appendicitis c) nephrolithiasis d) perforated colon cancer e) lymphoma f) horseshoe kidney g) diverticulitis h) ovarian cancer

a) periappendiceal abscess b) appendicitis d) perforated colon cancer e) lymphoma g) diverticulitis h) ovarian cancer NOT c) nephrolithiasis f) horseshoe kidney Given this patient's compelling history, acute appendicitis would probably be at the top of the list. Periappendiceal abscess could present with focal tenderness and a mass, although the time course from onset of symptoms to development of abscess is typically quite a bit longer than 24 hours. Particularly in the elderly patient, one must consider the possibility of a cecal cancer obstructing the appendiceal lumen resulting in appendicitis and a contained perforation. Acute sigmoid diverticulitis, while it usually presents with signs and symptoms localizing to the left lower quadrant, can mimic appendicitis when the sigmoid loop is redundant and lies in the right lower quadrant. Complications of locally advanced ovarian cancer can certainly have this clinical presentation as well. Horseshoe kidney, a congenital abnormality, is usually an incidental finding and does not present as an acute abdomen. Nephrolithiasis typically presents with flank pain and hematuria, without signs of inflammation. The pain associated with nephrolithiasis is typically cramping or intermittent in character.

A 65 year old black female smoker undergoes screening for abdominal aortic aneurysm with abdominal duplex ultrasound. She is found to have a 4.2 cm infrarenal aneurysm. She has a family history of coronary artery disease and diabetes. Which of the following was the most significant risk factor for her developing the aneurysm: a) smoking hx b) gender c) age > 50 d) race

a) smoking hx The likelihood of developing abdominal aortic aneurysm is influenced by multiple risk factors including older age, male gender, white race, positive family history, smoking history, hypertension, hypercholesterolemia, peripheral vascular disease and coronary artery disease. In this patient, her gender and race are not risk factors. A patient with a smoking history has a relative risk nearly 5 times greater than a non-smoker to develop AAA.

A 60-year-old man is evaluated for a 5-day history of lower abdominal pain and tenesmus. For the past 2 days, the abdominal pain has increased, and he has had nausea without vomiting. He has 2 previous episodes of similar symptoms that were treated on an outpatient basis. The patient has a history of chronic constipation that he manages with magnesium hydroxide. On physical examination, his temperature is 100.8°F (38.2°C), pulse is 90/min and regular, respirations are 16/min, and blood pressure is 140/90 mm Hg. No murmurs are heard. The lungs are clear to auscultation and percussion. The abdomen is distended with high-pitched tinkling bowel sounds. There is significant tenderness in the left lower quadrant with guarding but no rebound. Laboratory studies show a leukocyte count of 12,400/mm3 and hemoglobin of 12 g/dL. A nasogastric tube is placed and intravenous fluids are begun. Which of the following is the most appropriate next step in management? a) ex lap b) CT abd c) colonoscopy d) barium enema e) mesenteric angiography

b) CT abd The patient is most likely presenting with diverticulitis. Diverticulitis often presents with low grade temps, left lower quadrant pain and tenderness, and leukocytosis. The history of previous similar symptoms treated as an outpatient is also consistent with the diagnosis of diverticulitis. The next step in the management and to confirm the diagnosis (and rule out other possible diagnosis) is to obtain a CT scan of the abdomen. CT will also help to determine if the nature of the diverticulitis is complicated (such as perforation or abscess).

A 64 year-old man presents to your office with an incidentally identified abdominal aortic aneurysm (AAA) measuring 5.7 cm by screening duplex examination. He has mild COPD and a history of coronary artery disease, having had coronary stents several years ago after a myocardial infarction. He is able to climb 2 flights of stairs without difficulty. What is the next imaging study that should be obtained? a) no imaging is necessary, he should be booked for surgery b) computed tomographic angiography (CTA) c) CXR d) arteriography

b) CTA CT imaging allows the determination of both the size of the aneurysm and its relation to other intra-abdominal arteries as well as the length of proximal and distal landing zones that are necessary for evaluation of the possibility for an endovascular stent-graft. Ultrasound is user-dependent, frequently undersizes the aneurysm, and cannot frequently visualize the proximal extent of the disease. It is useful for screening and surveillance, but if repair is indicated, more advanced imaging is usually necessary. Angiography was historically used in evaluating patients for AAA, however with modern CT scanners providing three-dimensional reconstructions as well as excellent images for sizing, it has largely been replaced by CT scan in most centers.

Eight weeks after a 44-year-old man underwent laparoscopic adjustable gastric band placement, he comes to the emergency department because of a 4-day history of worsening nausea and pain after eating. He has been unable to tolerate any solid or liquid food for the last 2 days. On physical examination, pulse rate is 110/min and decreases to 85/min after administration of 2 L of intravenous crystalloid infusion. His vital signs are otherwise normal. The abdomen is soft and nontender to palpation. Laboratory studies show no abnormalities. An upper gastrointestinal series with water-soluble contrast shows no passage of contrast beyond the gastric band. Which of the following is the best next step in management? a) Operative revision of the band b) Deflation of his band by removing saline from the port c) Conversion to Roux-en-Y gastric bypass d) Nasogastric decompression with observation e) Upper endoscopy with balloon dilation of the stricture

b) Deflation of his band by removing saline from the port Inability to tolerate a diet after a laparoscopic band suggests either the band is too tight or has slipped. An UGI series or even plain X Rays can diagnose a slipped band but, if the band is too tight then an UGI series will be needed. Emergent treatment is to decompress the band by removing fluid from the port (band adjustment).

21 year old female comes in with 12 hours of pain that began around the umbilicus and migrated to the right lower quadrant. She has a fever of 101, a white blood cell count of 13 and right lower quadrant peritonitis on exam. Urine HCG and pelvic exam is negative. The most appropriate discussion to have with the patient next is: a) Discuss with the patient increased risk of infertility associated with appendectomy b) Discuss with the patient the risk of abscess formation associated with appendectomy surgery c) Discuss with the patient risks of C. diff colitis with antibiotics d) Discuss with the patient the risk of death associated with appendectomy surgery e) Discuss the risk of needing a temporary colostomy

b) Discuss with the patient the risk of abscess formation associated with appendectomy surgery This patient had a classic symptom complex and physical exam for appendicitis. It is not necessary to perform any other tests (such as a CT scan) and the proper action is to proceed to the operating room for an appendectomy. Her chance of fertility problems are higher if treatment for appendicitis is delayed, so A is incorrect. Although she will likely receive antibiotics for surgery, the risk for c.diff in a healthy female without prior hospitalization or exposure is extremely low and not the focus of the treatment, therefore I would not focus on it unless antibiotics were the only treatment planned. Although answer d. is partially correct, the risk of death for a 21 year old female, although not zero, is extremely unlikely with appendicitis surgery. In addition, the risk of death is higher from perforation of her appendix if surgery is not performed. Answer b. is the best answer because it addresses one of the more common complications associated with contaminated cases - organ space and wound infections.

A 51-year-old woman had a Roux-en-Y gastric bypass 10 years ago. She presents to the emergency department with 2 days of worsening abdominal pain, nausea, and vomiting. Her pulse in the emergency department is 120/min, and decreases to 115/min with administration of 2 L of Ringer's lactate solution. Her blood pressure remains unchanged at 90/65 mm Hg despite the fluids. On abdominal examination, there is moderate epigastric tenderness to palpation. Leukocyte count is 12,000/mm3, and hematocrit is 34%. Computed tomography of the abdomen shows gastric dilatation, mildly dilated proximal small bowel with a loop of mid jejunum that is more profoundly dilated and thickened followed by decompressed distal small bowel. The colon is decompressed. Which of the following is the most appropriate next step in management? a) Upper endoscopy with possible stent placement b) Emergent laparoscopy c) Mesenteric angiography d) Upper gastrointestinal contrast study e) Nasogastric tube decompression and serial abdominal examinations

b) Emergent laparoscopy This patient has a bowel obstruction and is quite dehydrated. This could be the result of an internal hernia and if not addressed promptly may result in ischemic bowel with disastrous consequences

A 28-year-old woman comes to your office with complaints of "hemorrhoids." She states that she has had intermittent bleeding for the past 3 months, and she reports "something is falling out of my rectum with every bowel movement and I have to push it back in." Her symptoms have not responded to topical hemorrhoid ointments. The patient has 3 children born by the vaginal route without complications. Her medical history is otherwise unremarkable. On physical examination, her temperature is 98.6°F (37.0°C), pulse is 60/min and regular, respirations are 12/min, and blood pressure is 100/60 mm Hg. Physical examination shows rectal mucosa that protrudes from the anus in the left lateral position when she strains. Which of the following is the most likely diagnosis? a) External hemorrhoids b) Internal hemorrhoids c) Rectal prolapse d) Rectal polyp e) Hypertrophied anal papilla

b) Internal hemorrhoids This young woman presents with prolapsing internal hemorrhoids. The intermittent bright red bleeding associated with prolapsing tissue is a common history. Hemorrhoids are commonly associated with a history of pregnancy and straining at defecation. There is usually minimal to no pain involved since the prolapsing internal hemorrhoids are within the rectal mucosa and receive enteric innervation. The dilated hemorrhoids and overlying rectal mucosa are susceptible to bleeding from straining and defecation. External hemorrhoids are not reducible. They are covered with squamous mucosa and are located under the anoderm in the anal canal. Anoderm has somatic innervation, and thrombosis and inflammation of external hemorhhoids causes moderate to severe pain. Bleeding from thrombosed hemorrhoids occurs when the clot erodes through the skin. External hemorrhoids that are not thrombosed are not likely to bleed. Prolapsed internal hemorrhoids consist of the submucosal tissue with the dilated hemorrhoids and the overlying rectal mucosa. There are three main columns of hemorrhoids - right anterior, right lateral and left posterior. One or all three of the columns may be enlarged and prolapse. Rectal prolapse refers to full thickness (mucosa, submucosa and muscularis) prolapse of the rectum through the anus. This presents as a concentric cone, and a small segment of rectum can prolapse as well as very large segments. Rarely, a pedunculated rectal polyp can prolapse and cause similar symptoms to a prolapsed hemorrhoid, but is easily distinguished on physical exam including anoscopy. Hypertrophied anal papillae occur at the dentate line and are usually caused from anal inflammation, crytotits and papillitis. Small hypertrophied papillae are common and associated with hemorrhoid disease. Occasionally one can get quit large and prolapse. They appear as a firm, tannish-white tissue, and are easily identified by palpation or direct visualization

48 year old male smoker with 30 pack year history with no additional significant past medical history is brought to the emergency room after a fall from scaffolding at work. On arrival, he was normotensive at 110/60, P: 80, with temperature of 37. On physical exam, he complained of leg pain on the left and the remainder of his physical exam including neurological evaluation was normal. His PA and lateral chest xray revealed no evidence of pneumothorax. He underwent CT scan of the abdomen and pelvis which revealed a 6.2cm left adrenal nodule. Laboratory results did not demonstrate functionality. What is the next step in management? a) Open adrenalectomy b) Laparoscopic adrenalectomy c) CT guided biopsy of adrenal mass d) Observation e) Rule out lung cancer by obtaining CT scan

b) Laparoscopic adrenalectomy This is a patient with history of smoking with an incidentally found adrenal mass on CT Scan. The risk of cancer increases with increasing size. At this point with lesion more than 6cm, it should be recommended that he undergo laparoscopic adrenalectomy. Although not clearly linked, there is an association between smoking and development of adrenal malignancy.

A 45-year-old woman who underwent laparoscopic Roux-en-Y gastric bypass 2 years ago comes to the physician because of a 1-month history of burning epigastric pain. She has not had nausea or vomiting during this time. Since the procedure, she has lost 84 lb (38 kg) and now has a body mass index of 25 kg/m2. Her medical history is also remarkable for a cholecystectomy 8 years ago. Her only current medication is ibuprofen, which she takes as needed for migraine headaches. On physical examination, temperature is 98.4°F (36.9°C), pulse is 70/min, respirations are 14/min, and blood pressure is 118/78 mm Hg. The abdomen is soft and nontender without any appreciable organomegaly. Which of the following is the most likely diagnosis? a) Gastic bezoar b) Marginal ulcer c) Internal hernia d) Chronic constipation e) Stomal stenosis

b) Marginal ulcer Patient undergoing a gastric bypass are prone to develop marginal ulcers and use of NSAIDS is thought to be a risk factor for ulceration. The epigastric burning pain is also suggestive of marginal ulceration.

A 24 year-old asymptomatic male was found to have a 2.5cm left adrenal mass on CT scan of the abdomen for abdominal pain. He was also found to have appendicitis and underwent laparoscopic appendectomy uneventfully. Upon discharge, you explain to the patient that he has a small adrenal nodule and he is given a follow up appointment. Which test would NOT be indicated to assess if this mass is a functional adenoma? a) Dexamethasone suppression test b) Plasma Testosterone Level c) Renin/Aldosterone level d) Serum electrolyte level e) Urinary and serum metanephrine/normetanephrine levels

b) Plasma Testosterone Level According to the NIH consensus on management of asymptomatic adrenal mass, the patient should be tested for renin/aldosterone level and serum electrolyte levels to rule out aldostrenoma; dexamethasone suppression test to rule out Cushing Syndrome and sub-clinical Cushing syndrome, and Urinary and serum metanephrine/normetanephrine levels to rule out Pheochromocytoma. Sex hormone levels should be ordered when virilization or feminization is apparent on patient presentation.

25 year old man presents with 18 hours of anorexia and peri-umbilical abdominal pain, now localized in the right lower quadrant. On exam he has a temperature of 101, HR 80, BP 124/82 and 02 saturation 98% on room air. His WBC is 14, the rest of his labs are normal. His CT scan shows "inflammation in the right lower quadrant - cannot differentiate acute appendicitis from sigmoid diverticulitis." The next step in his management should be: a) Order a water soluble enema to evaluate for diverticulitis b) Proceed urgently to the operating room for appendectomy c) Order blood cultures and admit for observation and antibiotics d) Discuss case with radiologist to see what additional testing would be helpful e) consider sending IBD (inflammatory bowel disease) blood panel

b) Proceed urgently to the operating room for appendectomy Although answer c is appropriate for treatment of diverticulitis, this patient has a classic constellation of symptoms for appendicitis and he is the wrong age for diverticulitis which usually affects older patients, therefore answer b is correct. Additional radiological studies are not necessary, so a. and d. are incorrect as well.

A 28 year old G2P2 female presents to the emergency department with 12 hours of worsening abdominal pain. The pain began and has remained in the right lower quadrant of her abdomen. She reports nausea, nonbloody diarrhea and 3 episodes of nonbilious emesis during this time. Her youngest child is 2 years old. On physical examination: Temperature: 38.8C HR 115 RR: 20 BP 85/60 SaO2 100% on room air Abdomen: Soft, tender in right lower quadrant with voluntary guarding. The remainder of her abdomen is nontender and not distended Which of the following elements from her history will be most useful in establishing the diagnosis? a) Symptoms of burning with urination b) Recently stopped oral contraceptive pills to attempt conception c) Recent history of travel d) Family history of inflammatory bowel disease e) History of treated sexually transmitted infection

b) Recently stopped oral contraceptive pills to attempt conception A ruptured ectopic pregnancy is a surgical emergency and can be difficult to distinguish from acute appendicitis. Although inflammatory bowel disease can mimic appendicitis, a positive family history in the absence of other symptoms to support this diagnosis would make it less likely. Urinary tract infection would certainly need to be excluded with urinalysis if dysuria is present, but dysuria can also present as a symptom of appendicitis due to secondary inflammation of the bladder. A history of a treated sexually transmitted infection may be useful in establishing the diagnosis, but only after the above, more urgent diagnoses have been excluded.

A 65-year-old man with recently diagnosed unresectable gastric cancer presents to the emergency department because of a 4-day history of nausea, non-bilious emesis, and epigastric pain. The patient has elected to not pursue chemotherapy treatment at this time. On physical examination, his temperature is 98.0°F (36.7°C), pulse is 130/min, respirations are 28/min, and blood pressure is 100/55 mm Hg. The mucus membranes are dry. Abdominal examination shows upper abdominal distention with tympany. No peritoneal signs are noted. Results of laboratory studies are most likely to show which of the following? a) lactic acidosis b) aciduria c) hyperchloremia d) hyperkalemia e) unconjugated hyperbilirubinemia

b) aciduria This patient has gastric outlet obstruction. The result is non-bilious vomiting which depletes hydrochloric acid from the stomach. This depletion of hydrochloric acid causes a hypocholemic metabolic alkalosis and dehydration because of the loss of H+ and Cl- ions in addition to fluid. The loss of protons triggers the kidney to preserve protons at the expense of potassium and thus hypokalemia ensues and the result is hypochloremic, hypokalemic metabolic alkalosis. With time, the renal compensation will continue and H+ will continue to be excreted in the urine, thus result in a "paradoxical aciduria".

In obtaining informed consent for appendectomy in a minor: a) The surgeon should minimize the complications of the procedure so as not to frighten the patient b) The girl's legal guardian must be involved in the process and must sign the consent c) Alternative options need not be discussed as the patient is a minor. d) The competency of the patient does not need to be of concern if the procedure is indicated e) Consent is not required as the procedure is an emergency f) Only the patient needs to sign the consent

b) The girl's legal guardian must be involved in the process and must sign the consent The risks, benefits, and alternatives of every procedure must be explained to all patients. If the patient is of legal age, the physician obtaining the consent must be convinced that the patient has "decisional capacity", meaning the ability to understand the particular decision being made. Psychiatric consultation should be sought if there is evidence of a chronic or acute psychiatric disorder, which may interfere with decisional capacity. All physicians should be proficient at determining decisional capacity. When at all possible the informed consent discussion must be documented and confirmed by signature of the patient or legal guardian, health care provider and a witness. Physicians should always act in the best interests of the patient in an emergency, life or death situation, when a health proxy is not available (e.g. usually next of kin -spouse, parent child- in that order), and when there are no known prior directives from the patient.

A 14 year old girl presents with a 12 hour history of lower abdominal pain, which has now localized to the right lower quadrant. She has noted anorexia, but no vomitting. She is sexually active, but uses birth control. Her last menstrual period was one week ago. On examination, the abdomen is flat. Bowel sounds are absent. There is rebound tenderness in the right lower quadrant. Vital signs include a blood pressure of 115/70 mmHg, a pulse of 102 per minute, respirations of 18 per minute, and a temperature of 38.0°C. Laboratory studies include a hemoglobin of 12.3, a WBC of 13,000 with a left shift. A beta HCG is negative. Which of the following would be a relative contraindication to a laparoscopic appendectomy? a) The patient has a periappendiceal abscess b) The patient has had multiple prior laparotomies c) The patient has Crohn's disease of the terminal ileum in addition to appendicitis d) Small umbilical hernia e) The appendix is perforated f) None of the above

b) The patient has had multiple prior laparotomies Previous abdominal surgeries may result in adhesions that would make laparoscopy either more challenging or prohibitive. Since there is no reliable way to know if adhesions would exist, save a history of severe adhesions found during surgery, this remains a relative counter-indication.

A 53-year-old woman who underwent laparoscopic gastric bypass 10 years ago is evaluated for memory loss and neuropathy of the both legs. Her husband reports that the patient has become increasingly forgetful, and she sometimes appears confused during ordinary daily activities. Her postoperative course was uncomplicated, and she has successfully maintained a weight of 110 lb, which corresponds to a body mass index of 20 kg/m². She has no history of serious illness except for episodes of chronic vomiting that she has learned to tolerate by restricting her diet. On physical examination, temperature is 98.6°F (37.0°C), pulse is 100/min, respirations are 14/min, and blood pressure is 110/70 mm Hg. Pedal edema is noted, but the extremities are warm and nontender. Neurologic examination shows symmetrically decreased sensation in the lower extremities that is more pronounced distally. The remainder of the physical examination is unremarkable. Which of the following is the most likely diagnosis? a) Calcium deficiency b) Thiamine deficiency c) Vitamin C deficiency d) Vitamin D deficiency e) Iron deficiency

b) Thiamine deficiency Chronic vomiting can predispose a patient to developing vitamin deficiency. Deficiency of Vitamin B1 causes disturbance of memory as well as peripheral neuropathy.

A 39-year-old woman (body mass index = 45 kg/m2) underwent a Roux-en-Y gastric bypass procedure 6 weeks ago and presents to the clinic for follow-up. She reports that she has been doing well with her liquid and then solid diet, but she has had progressive early satiety and postprandial nausea over the last 2 weeks. She does not have significant pain. Her current medications are a daily multivitamin and vitamin B12 and calcium supplements. On physical examination, her temperature is 98.6°F (37.0°C), pulse is 84/minute, respirations are 16/min, and blood pressure is 130/90 mm Hg. Her weight loss is appropriate for this postoperative time-point. Which of the following is the best next step in treatment? a) Placement of esophageal stent b) Upper endoscopy with balloon dilation of anastomosis c) Watchful waiting d) Placement of jejunal feeding tube e) Operative anastomotic revision

b) Upper endoscopy with balloon dilation of anastomosis These findings are consistent with stenosis of the gastrojejunostomy. Initial treatment of such stenosis is balloon dilation endoscopically.

A 39-year-old woman (body mass index = 45 kg/m2) underwent a Roux-en-Y gastric bypass procedure 6 weeks ago and presents to the clinic for follow-up. She reports that she has been doing well with her liquid and then solid diet, but she has had progressive early satiety and postprandial nausea over the last 2 weeks. She does not have significant pain. Her current medications are a daily multivitamin and vitamin B12 and calcium supplements. On physical examination, her temperature is 98.6°F (37.0°C), pulse is 84/minute, respirations are 16/min, and blood pressure is 130/90 mm Hg. Her weight loss is appropriate for this postoperative time-point. Which of the following is the most appropriate next step in management? a) Radionuclide gastric emptying study b) Upper gastrointestinal contrast study c) Diagnostic laparoscopy d) Plain radiograph of the chest e) Computed tomography of the abdomen

b) Upper gastrointestinal contrast study The inability to tolerate a diet that has progressed from solids to liquids in an otherwise stable patient is very suggestive of a stenosis of the gastrojejunostomy. his is easily diagnosed on an UGI series. Endoscopy can also visualize the stenosis and can also be used to treat the stenosis using balloon dilators.

A 36-year-old otherwise healthy man presents to the emergency room with a 24-hour history of diffuse abdominal pain, obstipation, nausea and emesis. His past medical history is significant for an open appendectomy at the age of 5 years. On examination, temperature is 99.6°F (37.6°C), pulse is 110/min, respirations are 22/min, and blood pressure is 110/70 mm Hg. The abdomen is distended, bowel sounds are high pitched and metallic, percussion is tympanitic, and there is tenderness to palpation over all 4 quadrants. Which of the following is the most likely cause of his obstruction? a) intussusception b) adhesions c) malrotation d) cancer e) internal hernia

b) adhesions The options presented are causes of bowel obstruction. In a patient with an abdominal surgical history, adhesions are the most likely case of the obstruction even if the surgery was minor.

A previously healthy 22-month-old boy is brought to the emergency department because of paroxysms of severe colicky abdominal discomfort that has persisted for 24 hours. His mother reports that he passed maroon-colored stool earlier today. He has not had fever. The patient's temperature is 98.6°F (37.0°C), pulse is 90/min, respirations are 24/min, and blood pressure is 105/80 mm Hg. On cardiac examination, S1 and S2 are normal, and no murmurs are heard. The lungs are clear to auscultation. Abdominal examination shows distention and right-sided fullness. Laboratory studies show a leukocyte count of 7800/mm3. An abdominal radiograph shows dilated loops of small bowel. Which of the following is the best next step in diagnosis? a) colonoscopy b) barium enema c) CT abdomen d) surgical exploration e) upper GI series with water-soluble contrast

b) barium enema The 22 month old boy's symptoms are most consistent with intussusception. These include colicky pain, maroon-colored stools, and paroxysmal nature of pain. Of the diagnostic tests listed, barium enema is the most successful in achieving the diagnosis, but also can be therapeutic as well.

An 80-year-old woman is evaluated for a 2-day history of nausea, vomiting, and abdominal distention and pain. Her medical history is remarkable for type 2 diabetes mellitus treated with metformin. She resides in an assisted living facility where she has fallen 3 times in the past year. On physical examination, her temperature is 98.6°F (37.0°C), pulse is 70/min and regular, respirations are 16/min, and blood pressure is 140/60 mm Hg. The abdomen is distended with diffuse tenderness and tympany on percussion. While internally rotating the flexed right hip, she complains of pain extending down the medial aspect of the right thigh. Which of the following is the most likely diagnosis? a) acute appendicitis b) obturator hernia c) cecal carcinoma d) psoas abscess e) pelvic fx

b) obturator hernia The patient is presenting with a bowel obstruction. In a patient with no an abdominal surgical history, adhesions are uncommon cases should be entertained. The physician exam is consistent with a strangulated hernia, obturator hernia in this case.

With a highly suspected diagnosis of sigmoid volvulus, which of the following is the best next step in management? a) NG tube with IV hydration b) sigmoidoscopy c) mesenteric angiography d) saline enema e) CT abdomen

b) sigmoidoscopy Sigmoid volvulus is a form of large bowel obstruction that is ideally decompressed prior to definitive management (surgery to avoid recurrence). While decompression has been demonstrated with barium enema examination, sigmoidsocopy (rigid or flexible) has the best chance of decompression. Decompression allows for time to resuscitate, and possibly bowel prep the patient, but surgical management is still recommended (preferably during the same hospitalization) since eventual recurrence of the volvulus is high.

A 65 year old male presents to the emergency room with 12 hours of right lower quadrant pain, leukocytosis, and anorexia. His pain began as periumbilical, vague pain and progressed over the last 8 hours to focal pain in the right lower quadrant. It is during this time period that he lost his appetite. He does not feel ill. Has no fever. On physical exam, he has point tenderness and guarding two-thirds of the distance from his umbilicus to his right anterior superior iliac spine. Based on this history and physical exam, he is diagnosed with typical acute appendicitis and is taken to surgery for laparoscopic appendectomy. The surgeon describes the appendix as normal appearing. Appendectomy is performed without complication. Final pathological analysis of his surgical specimen shows a normal appendix. The rate of misdiagnosis, or negative appendectomy, in this situation is best described as: a) 5% b) 15% c) 65% d) 50% e) 35%

c) 65% Patients presenting with a history, laboratory values, and exam typical of acute appendicitis may be taken to surgery without imaging. These patients are most often males in the second through forth decade of life with a white cell count 10-18K. Alvarodo score may be used to evaluate typical history for appendicitis. Despite imaging techniques employed, misdiagnosis will occur. Females and those age 65 and older have increased rates of negative appendectomy.

A 25 year old male is seen in the emergency room with focal right lower quadrant pain. He had mild abdominal discomfort last night which he has difficulty specifically describing. This morning, he had severe right lower quadrant pains which he says are much worse. He has no fever. His WBC is 14k with 90% PMNs. CT scan of the abdomen and pelvis identifies an inflamed appendix with an appendicolith. He is evaluated by a surgeon who confirms the diagnosis of appendicitis and he is taken to surgery for appendectomy. He is found to have typical, non-perforated appendicitis. Which of the follow best describes the appropriate use of antibiotics in this patient? a) Antibiotics should be continued post-operatively until the patient's WBC returns to normal b) Antimicrobials are not necessary for non-perforated appendicitis c) Antibiotics should be given pre-operatively and for no more than 24 hours post-operatively d) A single dose of antibiotic pre-operatively is necessary e) Antibiotics should be given orally

c) Antibiotics should be given pre-operatively and for no more than 24 hours post-operatively Several studies have demonstrated the efficacy of perioperative antibiotics in lowering the infectious complications of appendicitis. Preoperative antibiotics are universally accepted. Patients with acute appendicitis should have antimicrobials continued for not more than 24 hours after surgery. If perforated or gangrenous appendicitis is found, antibiotics are continued for a longer course - typically until systemic signs of infection (eg, WBC elevation, fever) are resolved

A 24 year old woman presents to the office with a fever seven days following a laparoscopic appendectomy. She doesn't feel well and complains of mild nausea. Vital signs include a blood pressure of 140/75 mmHg, a pulse of 102 per minute, respirations of 16 per minute, and a temperature of 38.5° C. On physical examination, the wound is well healed. There is tenderness in the RLQ without rebound. A digital rectal examination and bimanual examination of the pelvis suggest the presence of pelvic tenderness. There is no evidence of a palpable mass. The next most appropriate step is: a) send the patient home with follow-up in one week b) admit the patient for observation c) CT d) CXXR e) flat and upright films of the abdomen f) IV antibiotics

c) CT CT scan is the best way to assess for the presence of an intra abdominal abscess. If an abscess is documented, IV antibiotics should be started and a percutaneous drainage procedure, if possible, should be performed. Observation alone, whether at home or in the hospital is inadequate care.

A 22 year old female with clinical suspicion for hypercortisolism is seen by the endocrinologist for follow up after obtaining laboratory work. In addition to her Cushinoid features, all of the lab results are consistent with hypercortisolism. She is now in your office to discuss the results and the next step in management. Her laboratory results are as follows.: Adrenocorticotropic hormone: low Urine free cortisol: Elevated Cortisol (8 AM) After 1 mg of dexamethasone the night before: Elevated After 8 mg of dexamethasone the night before: Elevated What test do you order next? a) CT scan of the head b) Serum/urine catecholamine c) CT Scan of the abdomen d) Cosyntropin stimulation test e) MRI of the head

c) CT Scan of the abdomen This is a patient with suspected hypercortisolism. With low ACTH and cortisol levels that are not suppressed after high and low dose dexamethasone suppression test, one of the suspicions is that the patient has an ectopic adrenal nodule that is responsible for the results shown above. On the contrary, patients who have pituitary adenomas resulting in Cushings will have elevated ACTH, elevated urine free cortisol, and high dexamethasone suppression test will partially depress the cortisol level. Thus, in this patient, one of the areas to evaluate is the adrenal gland by obtaining CT Scan of the abdomen.

A 38-year-old woman comes to your office because of a 4-week history of painful bowel movements associated with a small amount of blood on the toilet tissue with wiping. She has no history of similar episodes but is very concerned because her grandfather died of colon cancer. She has occasional constipation but has otherwise been healthy. The patient is in a monogamous relationship with her husband. With the exception of topical hemorrhoid ointment, she takes no medications. On physical examination, her temperature is 98.6°F (37.0°C), pulse is 80/min and regular, respirations are 14/min, and blood pressure is 110/70 mm Hg. Physical examination shows a posterior midline skin tag that is tender to palpation. Digital rectal examination cannot be completed because of severe pain and sphincter spasm. Which of the following is the most likely cause of this patient's symptoms? a) Perianal abscess b) Condyloma accuminata c) Chronic anal fissure d) Thrombosed hemorrhoid e) Squamous carcinoma of anus

c) Chronic Anal Fissure This patient has a classic history for a chronic anal fissure. Minor anal trauma from constipation (usual history) or diarrhea can lead to an acute anal fissure. The patient may not have pain initially but may see bright red blood on the toilet tissue. Pain may occur with repeated episode. Initially the pain is with defecation. With time the pain may be initiated by defecation but lingers long afterward due to sphincter spasm which leads to ischemia of the anoderm which in turns leads to a chronic ulcer and scarring into the internal anal sphincter. Patients avoid defecating due to the pain leading to more constipation and propagation of the problem. Chronic anal fissures are usually located in the posterior midline, the least distensible area of the sphincter mechanism. Pain and sphincter spasm makes it very difficult to do a digital rectal exam without anesthesia. There is often a prominent skin tag or sentinel pile that is distal to the fissure that lies in the anal canal. This may appear edematous and be painful when palpated, and is often confused as a thrombosed hemorrhoid or anal warts (condyloma accuminata) by inexperienced clinicians. Carefully spreading the anal skin and sphincter will reveal the small ulcer proximal to the tag, but this can be difficult to accomplish without anesthesia in many patients. Thrombosed hemorrhoids have an acute onset and the patient can usually feel a nodule adjacent to the anal canal. Usually one sees a blue-black colored nodule that is tender to touch. There usually is no evidence of sphincter spasm or antecedent history of blood on the tissue. A perianal abscess will also cause pain, and the patient often can feel a tender lump in the area. Defecation may be painful with an abscess, since these usually start as an anal crypt abscess and can communicate through a portion of the sphincter to the subcutaneous perianal tissue. These may appear as a fluctuant area with erythematous overlying skin or as just a tender indurated area. Intramuscular abscesses show no perianal skin changes, but they usually are associated with very painful digital rectal exams. Condyloma accuminata or anal warts due to HPV infection appear as solitary or multiple papules that sometimes can be quite large and bulky. They are usually associated with less severe pain but bulky warts can become very painful. Anal warts are associated with a risk for developing squamous cell carcinoma. Squamous cell carcinoma can occur in and around the anal canal. These are often painful and can be confused with chronic fissures. Anal cancers can occur anywhere around the anus and a chronic ulcer that is not in the posterior midline should always be suspicious for cancer. Chronic posterior midline fissures that do not respond to treatment should be biopsied to rule out malignancy.

A previously healthy 27-year-old man comes to the emergency department because of perianal pain, swelling, and drainage of a small amount of pus for the past 2 weeks. He reports a 10-pound (4.5-kg) weight loss, abdominal cramping, and intermittent mucus in his stools. He reports having 7 to 8 stools per day. His only medication is loperamide. On physical examination, his temperature is 99.0°F (37.2°C), pulse is 88/min and regular, respirations are 14/min, and blood pressure is 120/80 mm Hg. The abdomen is flat and soft with mild tenderness to palpation over the right lower quadrant. Rectal examination shows a chronic-appearing fistula-in-ano. Which of the following is the most likely diagnosis? a) Colorectal cancer b) Ulcerative colitis c) Crohn's disease d) Squamous carcinoma of anus e) Irritable bowel disease

c) Crohn's disease This patient has a classic history for Crohn's disease, one of the chronic inflammatory bowel diseases. Chronic diarrhea, cramping and mucus in the stool is a finding often associated with either Crohn's disease or ulcerative colitis. Ulcerative colitis (UC ), is characterized by mucosa- based inflammation and always involves the rectum with varying degrees of proximal extension in the colon. Perianal disease and fistulas are usually not associated with UC. Crohn's disease is characterized by full thickness inflammation in the GI tract. There can be areas of normal tissue between areas of inflammation (skip disease) and rectal sparing is common. Perianal disease, (including abscesses, fissures and fistulae) is common with Crohn's disease, and some patients develop severe disease with multiple fistulae (watering-can anus). Patients with colorectal cancer often are asymptomatic. Gross rectal bleeding without diarrhea can be a sign of a distal colorectal cancer. Obstructive symptoms are more common, and can be associated with paradoxical diarrhea, but this is usually less severe and not associated with significant amounts of mucous. Perianal disease is usually not associated with colorectal cancer. Patients with colorectal cancer are usually older unless there is a genetic risk such as with familial polyposis. Perianal squamous cell cancers can occur in young individuals, particularly those with a history of HCV infections. The other symptoms manifested in this patient are rarely present. Diarrhea can be a prominent symptom in some patients with irritable bowel syndrome (IBS), but perianal disease is not a manifestation of IBS.

A 29-year-old woman had Roux-en-Y gastric bypass 6 weeks ago. Her postoperative course has so far been uncomplicated. She presents to the emergency department complaining of diaphoresis, abdominal pain, nausea, and dizziness. Her symptoms started soon after eating dinner. On physical examination, her temperature is 98.6°F (37.0°C), pulse is 90/min, respirations are 18/min, and blood pressure is 110/80 mm Hg. The abdomen is soft and nontender to palpation. Computed tomography of the abdomen shows no abnormalities. Which of the following is the most likely diagnosis? a) Internal Hernia b) Marginal Ulcer c) Dumping syndrome d) Anastomotic leaks e) Panic attack

c) Dumping syndrome After a gastric bypass, patients become prone to suffering from postgastrectomy syndromes one of which is the dumping syndrome. The rapid emptying of a carbohydrate load from the stomach into the small bowel casues vasomotor and hormonal changes that often present with features described above.

An 80-year-old woman who resides in a nursing home is brought to the emergency department for a 2-day history of abdominal distention and obstipation. She has no history of previous surgeries. Her current medications are ibuprofen and omeprazole. On physical examination, her temperature is 98.6°F (37.0°C), pulse is 80/min and regular, respirations are 24/min, and blood pressure is 149/80 mm Hg. No murmurs are heard, and the lungs are clear to auscultation and percussion. The abdomen is markedly distended and tympanitic with diffuse tenderness but no rebound. Rectal examination shows no stool. Plain abdominal x-ray shows a markedly distended, coffee bean-shaped loop of bowel with haustral markings in the right upper quadrant. Which of the following is the most likely diagnosis? a) Gallstone ileus b) SBO c) Sigmoid volvulus d) Diverticulitis e) Ogilve's pseudo-obstruction

c) Sigmoid volvulus The clinical picture and exam are most consistent with a bowel obstruction. The markedly distended abdomen is typically seen when a large bowel obstruction is present, but small bowel obstruction is still possible. The radiological workup (x-rays) are classic for a large bowel obstruction due to sigmoid volvulus. The clinical and x-ray picture are not consistent with Ogilve's pseudo-obstruction.

A 40-year-old woman who underwent retrocolic laparoscopic gastric bypass 12 months ago presents to the emergency department with a 10-hour history of severe intermittent cramping pain in the mid abdomen that is exacerbated by meals. She also has nausea and vomiting, and she is now unable to tolerate liquids. At her most recent physical examination 3 weeks ago, the patient weighed 126 lb (57 kg), a 115-lb (52-kg) decrease from baseline before surgery. On physical examination in the emergency department, temperature is 98.6°F (37.0°C), pulse is 90/min, respirations are 16/min, and blood pressure is 98/68 mm Hg. The mucous membranes are dry. The mid abdomen is tender to palpation; there is no rebound tenderness or guarding. Results of laboratory studies are all normal. Plain films of the abdomen and pelvis shows dilated loops of small intestine and a dilated excluded stomach. Intravenous fluids are initiated. Which of the following is the best next step in management? a) Upper endoscopic management b) Observation with nasogastric decompression c) Emergent diagnostic laparoscopy d) Abdominal ultrasound e) Proton pump inhibitor therapy

c) Emergent diagnostic laparoscopy In a patient with gastric bypass, dilated loops of bowel with epigastric tenderness and vomiting is highly suggestive of a bowel obstruction due to an internal hernia. Delay in management could lead to ischemic bowel and /or perforation with significant disability or death.

A previously healthy 21-year-old man comes to the emergency department with severe anal pain that began 24 hours ago and has worsened during the past 3 hours. He takes no medications. On physical examination, his temperature is 99.0°F (37.2°C), pulse is 105/min and regular, respirations are 16/min, and blood pressure is 120/80 mm Hg. Examination of the perineum shows no abnormalities. Rectal examination cannot be completed because of severe pain. Which of the following is the best next step in management? a) Antibiotics for 7 days b) Application of topical xylocaine jelly c) Examination under anesthesia in operating room d) Computed tomography of the pelvis e) Endorectal ultrasound

c) Examination under anesthesia in operating room Acute anal pain should always initiate an examination and not be treated over the telephone. This patient's history and physical findings are highly suspicious for an intramuscular perirectal abscess. A thorough examination is necessary, and because of pain should be performed urgently under anesthesia in the operating room with the patient consented to drain an abscess or do other measures to obtain the diagnosis and treat the problem. Empiric therapy with antibiotics alone is not appropriate for a perirectal abscess which also needs to be drained. An urgent examination is necessary to confirm the diagnosis. Computed tomography (CT) scans should be done only to investigate a problem identified on a complete examination, and should not be done first to rule out disease. Endoscopic ultrasonography should never be done before a complete anorectal examination. Topical anesthetics such as lidocaine (Xylocaine) jelly are usually inadequate to control pain in order to do a complete anorectal examination. Treating the pain with topical anesthetics without determining the cause of the pain is not appropriate.

A 38-year-old man (body mass index = 68 kg/m2) has had vague abdominal discomfort since undergoing a difficult laparoscopic gastric bypass operation 3 days ago. During surgery, a methylene blue test demonstrated no leakage. Today, his Jackson-Pratt drain is suspicious for food coloring from his liquid diet. An upper gastrointestinal series shows a leak of water-soluble contrast into the Jackson Pratt drain. On physical examination, temperature is 98.4°F (37.0°C), pulse is 96/min, respirations are 18/min, and blood pressure is 120/78 mm Hg. Urine output is 30 mL/hr. His abdominal discomfort has improved slightly. Complete blood count shows a leukocyte count of 10,800/µL. Which of the following is the best next step in management? a) Continue oral liquids b) Diagnostic laparoscopy and repair of leak c) Insertion of central line for total parenteral nutrition d) Nasogastric tube insertion e) Immediate exploratory laparotomy and repair of leak

c) Insertion of central line for total parenteral nutrition This patient has a leak but is hemodynamically stable and the JP drain is managing the leak by draining stomach contents. Under such circumstances the patient can be managed non-operatively by not feeding them by mouth, leaving the drain in place and allowing the leak to heal. If the patient develops peritoneal signs, has a free leak into the peritoneal cavity or develops hemodynamic instability an operation will become necessary.

A 52 year old man with a 6 centimeter aneurysm has elected to undergo repair with an endovascular stent-graft. What are the downsides to this repair? a) Risk of developing an endoleak, increased risk of perioperative complications b) Increased risks of perioperative complications, requirement of appropriate anatomy c) Need for frequent CT scanning postoperatively, requirement of appropriate anatomy d) Need for a large abdominal incision, inability to use spinal anesthesia e) Need for frequent CT scanning postoperatively, need for a large abdominal incision

c) Need for frequent CT scanning postoperatively, requirement of appropriate anatomy Endovascular aneurysm repair (EVAR) is a minimally invasive approach to aneurysms. This requires an aneurysm with permissive anatomy, having an adequate length and diameter of proximal landing zone (neck) below the renal arteries as well as a distal landing zone in the iliac arteries. Small incisions are made in the groin, or the procedure is done entirely percutaneously via groin punctures, obviating the need for a large abdominal incision. A lighter anesthesia is required, and this can procedure can be done under spinal block and local anesthesia if needed. Recovery is typically short, with most patients being discharged home on post-operative day 1 or 2. Postoperatively, however, patients are required to follow up regularly with surveillance imaging including CT scans initially to ensure no endoleaks (flow of blood outside of the stent-graft) have occurred. This may occur in up to 20% of patients. If there are several consecutive CT scans with no endoleak demonstrated, duplex ultrasound may be an option for continued surveillance.

One day after undergoing laparoscopic gastric bypass, a 29-year-old woman is evaluated for severe tachycardia and left shoulder pain. The procedure was uncomplicated. The patient has a 10-year history of anxiety. On physical examination, temperature is 100.5 °F (38°C), pulse is 130/min, respirations are 28/min, and blood pressure is 106/74 mm Hg. The surgical site appears unremarkable. Which of the following is the best next step in management? a) Radiograph of the left shoulder b) Observation and antibiotic therapy c) Upper gastrointestinal series with water-soluble contrast d) Intravenous anti-anxiety medication e) Upper endoscopy and proton pump inhibitor therapy

c) Upper gastrointestinal series with water-soluble contrast In this patient a leak must be excluded using either an UGI series or a CT scan with oral contrast.

A 55 year-old man is evaluated for a 10 month history of recurrent episodes of sweating, palpitations, and headaches. He otherwise has an unremarkable medical history. He is currently taking Atenolol and Lisinopril to control his blood pressure. Physical exam revealed a very anxious appearing man with sweaty palms and neck exam significant for normal thyroid glands. His vital signs include a temperature of 37.8 C, blood pressure of 162/95, pulse rate of 90bpm and respiratory rate of 16 breaths per minute. Laboratory studies revealed elevated spot plasma metanephrine levels. He subsequently undergoes CT scan of the abdomen and pelvis demonstrates a 5mm nodule within the right adrenal. MRI of the abdomen reveals a similar finding, otherwise negative. What is the next best step in the management of this patient? a) Laparoscopic right adrenalectomy b) PET Scan c) Venography with venous sampling bilateral Adrenals d) MIBG Scan e) Ultrasound of the abdomen

c) Venography with venous sampling bilateral Adrenals This patient has signs and symptoms of pheochromocytoma. Biochemically it was shown that he has elevated metanephrine levels consistent with diagnosis of pheochromocytoma. However, neither the CT scan of the abdomen nor the MRI of the abdomen could be considered definitive localization as the ? mass seen is very small and could be an incidental adenoma or even a false positive read (since so small). The next test to look for adrenal mass is the MIBG scan. This is a test that utilizes a radioactive isotope that is selectively taken up by tissues that secrete catecholamines. MIBG scan is usually reserved for patients where the CT scan finding is equivocal or to diagnose extra-adrenal pheochromocytoma. In this case it would not be helpful as even if this 5mm mass is the lesion, it is too small to pick up by nuclear scan and also it is within the adrenal, so not helpful as adrenals will have a normal physiologic uptake by MIBG (that is why it is helpful to assess an extra-adrenal mass). Venography with venous sampling of bilateral adrenal glands may be useful when dealing with bilateral adrenal masses and for planning surgical resection to see whether a nodule is biochemically active. The downside to the test is that it is invasive and should be reserved after non-invasive testing has been attempted.

A 24 year old man undergoes laparoscopic appendectomy for a gangrenous, non perforated appendicitis. He received 1 gram of cefoxitan IV preoperatively. Postoperatively, he is awake and alert; he is tolerating a diet. On examination, the abdomen is not distended. Bowel sounds are present. There is generalized, mild abdominal tenderness to palpation. Vital signs include a blood pressure of 115/70 mmHg, a pulse of 85 per minute, respirations of 18 per minute, and a temperature of 37.5°C. Which of the following is the most appropriate plan for antibiotic management? a) continue for 48 hours after surgery b) continue until the white blood cell count (WBC) normalizes c) antibiotics should not be continued post-operatively d) continue for one week e) antibiotics should be continued until intraoperative cultures have been reported

c) antibiotics should not be continued post-operatively Antibiotics should be started within one hour prior to surgical incision to prevent wound infection. Following removal of the inflamed organ, there is no indication to continue antibiotics, certainly not for longer than 24 hours post-operatively.

You are asked to evaluate a 74-year-old woman who was admitted to the hospital 16 hours ago because of nausea, bilious emesis, and abdominal distention that persisted for 24 hours. She has moderate hypertension that is well controlled with a calcium channel blocker. Her surgical history is remarkable for an appendectomy at age 19 years and total abdominal hysterectomy for uterine leiomyoma at age 47 years. On physical examination, her temperature is 98.8°F (37.1°C), pulse is 70/min, respirations are 14/min, and blood pressure is 130/65 mm Hg. The abdomen is tympanitic to percussion. There is minimal tenderness to palpation in the epigastrium, and rectal tone is normal. A nasogastric tube is draining clear bilious material. Which of the following is the most appropriate next step in management? a) upper GI series b) colonoscopy c) continued observation d) diagnostic laparoscopy e) placement of rectal tube

c) continued observation The patient is in the hospital with a picture of small bowel obstruction probably due to adhesive disease (with her history of abdominal surgery). With an improving and stable abdominal example, continued non-operative management is warranted since the majority of patients (~80%) will continue to improve and avoid any invasive procedures. Continued close monitoring is important however as surgical intervention maybe required.

Examination of the abdomen in a patient suspected of having appendicitis begins with: a) light palpation b) right heel tap c) inspection d) auscultation e) deep palpation

c) inspection As with an examination for any purpose, physical exam should be done the same way in every patient and should always begin with inspection. The abdomen is exposed and thoroughly inspected for evidence of old surgical scars, distention, symmetry, masses, visible peristalsis, hernias, and pulsations, any of which may be associated with an acute abdomen. Inspection is followed by auscultation, then light palpation, deep palpation and examination for special signs. (Note that percussion, while useful for a general orientation to a non-tender abdomen, will be painful for the patient with peritonitis and should not be routinely performed before light palpation.)

A 20 year old man presents with a 24 hour history lower abdominal pain, which has localized to the right lower quadrant. The patient has been nauseated, and has had little to eat or drink. On physical examination the abdomen is tender in the right lower quadrant with rebound tenderness. Vital signs include a blood pressure of 120/80 mmHg, a pulse of 105 per minute, respirations of 18 per minute, and a temperature of 37.9° C. Which of the following findings on urinalysis would be most consistent with this patient's history? a) 20 RBCs per HPF on UA b) 1+ protein c) specific gravity 1.026 d) white cell casts e) 3+ glucose

c) specific gravity 1.026 While a "pelvic appendicitis" (an inflamed appendix that happens to be located in the pelvis) may cause presence of WBCs in a urinalysis, the urinalysis is typically normal in patients presenting with appendicitis. This patient's presentation does provide any reason for the presence of glucose, blood, casts, or protein in the urine. An elevated specific gravity can be associated with nausea and dehydration.

exclude in RLQ ddx

cystitis, ruptured graafian follicle, ovarian torsion, perforated peptic ulcer disease, femoral venous thrombosis Genitourinary infections cause lower abdominal pain and ileus, but are usually distinguished by the urinalysis. Ovulation with rupture of a graafian follicle (otherwise known as mittelschmerz) can cause brief abdominal pain and if on right side, can be confused with appendicitis. Femoral venous thrombosis would most commonly present with lower extremity pain and swelling.

A previously healthy 12 year old girl presents with a six hour history of a dull, aching, peri-umbilical pain, associated with a loss of appetite. She vomited once an hour ago. Vital signs include a blood pressure of 110/70 mmHg, a pulse of 102 per minute, respirations of 18 per minute, and a temperature of 100.8° F. You are concerned the patient has early appendicitis and order laboratory studies. Which laboratory findings would be consistent with the diagnosis of early acute appendicitis? a) >20 RBCs per HPF in urine b) elevated serum lipase c) >20 WBCs per HPF in urine d) 12 K WBCs e) Serum total bilirubin of 7mg/dL

d) 12 K WBCs Laboratory results need to be interpreted in light of the patient's clinical picture. While later/complicated appendicitis can have a wide spectrum of presentations, early appendicitis is typically seen with a mildly elevated WBC. While a few WBCs and/or RBCs can be seen in the urine in association with appendicitis, especially if the appendix is in the pelvis, these tend to number less than 5 cells per HPF. Elevated lipase or bilirubin is not typically seen in early appendicitis.

A patient arrives to the emergency room complaining of right lower quadrant pain which was present when they awoke this morning and has remained constant. No other symptoms are reported. The WBC count is 16k with 89% PMNs. Temperature is 37.9C. All other lab values are normal. Abdominal exam finds focal tenderness in the right lower quadrant with guarding and rebound tenderness. This tenderness, although somewhat less, is reproduced with deep palpation and release of the left side of the abdomen. This history and exam, in which of the following patients, warrants immediate surgery and no further testing. a) A 28 year old male who is morbidly obese and vomited once early this morning and is now hungry b) A 25 year old male who reports being hungry despite his pain. c) A 28 year old female who has a history of Crohn's enteritis. d) A 31 year old male who tells you vomited once early this morning and remains anorexic. e) A 27 year old female who is married and reports her LMP was about two weeks ago.

d) A 31 year old male who tells you vomited once early this morning and remains anorexic. The diagnosis of appendicitis should be questioned if the patient is not anorexic. In age appropriate females, pain with ovulation (ruptured graafian follicle, so called mittelschmerz) may mimic appendicitis. Vomiting is very common in appendicitis but typically ceases after one or two episodes. Crohn's enteritis may present with symptoms similar to acute appendicitis

A 42 year-old male undergoes a CT scan of the chest, abdomen, and pelvis following a motor vehicle crash and is found to have a 3cm left adrenal nodule. What is the most appropriate next step in management? a) CT guided core needle biopsy b) No further work-up is indicated c) Repeat CT scan in 3 months d) Basic metabolic profile and 24 hour urine samples e) Laparoscopic left adrenalectomy

d) Basic metabolic profile and 24 hour urine samples Work-up of the adrenal incidentaloma is an important part of any physicians knowledge base. Any adrenal nodule first requires evaluation of whether it is functional or non-functional. If it is non-functional, evaluation for any potential malignancy producing an adrenal metastasis is warranted. For non-functional adrenal nodules less than 4cm, interval follow-up via CT scan to monitor for growth of the nodule is indicated. Functional nodules, or non-functional nodules 4cm or larger should be surgically removed. CT guided core biopsy should only be reserved to document a metastatic lesion to the adrenal gland after the primary malignancy has been identified AND a pheochromocytoma has been ruled out.

17 year old male high school sprinter, presents to the emergency room with worsening right lower quadrant abdominal pain and nausea over the last 24 hours. He had a documented fever of 101 at home. Currently, his vital signs are normal except for a heart rate of 105 and a temperature of 100.5. He has right lower quadrant peritonitis on exam at McBurney's point. White blood cell count is 13,000 and CT scan is indeterminate for appendicitis due to non-visualization of the appendix. The next step should be: a) Discharge the patient for follow up with his primary care with a diagnosis of gastroenteritis b) Obtain consultation from Gastroenterology c) Ask the lab to perform a differential on the white blood cell count d) Consent the patient and mother for an appendectomy e) Admit the patient for observation and serial exam

d) Consent the patient and mother for an appendectomy This is a challenging case because imaging does not necessarily support the working diagnosis. The history and physical exam is consistent with appendicitis while the CT scan is not. Answer a. is incorrect because you cannot discharge a patient who has abdominal pain and peritonitis - gastroenteritis never causes peritonitis. Answer b. is also problematic since it will lead to delay to diagnose a surgical problem. Although answer c. may provide more information, it does not represent a decision for treatment so it is not the best answer. The decision between d. and e. is based on the fact that the patient has focal peritonitis, is a 17 year old male with a classic history and physical exam for appendicitis and should undergo an appendectomy. There are no "perfect" radiographic studies for appendicitis especially in thin patients (sprinter), the appendix can be obscured by adjoining bowel loops and paucity of intra-abdominal fat. Finally, in a patient with peritonitis, it is necessary to act quickly unless there is a definitive diagnosis and plan. It is always safer to perform a negative appendectomy in this setting than to have an appendix perforate while the patient is being observed.

A 25-year-old man comes to the emergency department because of a 4-week history of increasingly severe bloody diarrhea and abdominal pain without fever or chills. He has been obstipated over the past 24 hours. He has had 2 previous episodes of less severe symptoms that were treated on an outpatient basis. With the exception of an appendectomy at the age of 12 years, he has no other significant medical history and he takes no medications. On physical examination, his temperature is 97.7°F (36.5°C), pulse is 90/min, respirations are 22/min, and blood pressure is 130/90 mm Hg. The abdomen is distended and mildly tender in the right lower quadrant without guarding or rebound. Laboratory studies show a leukocyte count of 11,300/μL and hemoglobin of 12 g/dL. Which of the following is the most likely diagnosis? a) adhesive SBO b) Colon CA c) Yersinia enterocolitis d) Crohn's e) Meckel's diverticulitis

d) Crohn's The progressive symptoms and visible blood in the stool in a 25 year old are concerning for Crohn's disease. While adhesive small bowel obstruction can result in abdominal pain and tenderness, it does not present with bloody diarrhea. Meckle's diverticulitis may have a similar presentation, but very unlikely in this age group.

A 72-year-old woman is brought to the emergency department for a 24-hour history of nausea and abdominal fullness. Her last bowel movement was 2 days ago; she has had no blood in her stool. At the age of 49 years, she underwent cholecystectomy and total hysterectomy for uterine cancer, followed by pelvic radiation therapy. She has a 20-year history of rheumatoid arthritis treated with prednisolone, 5 mg daily. On physical examination, her temperature is 98.4°F (36.9°C), pulse is 66 beats/min, respirations are 14/min, and blood pressure is 120/58 mm Hg. The abdomen is distended without guarding or rebound tenderness. Bowel sounds are increased and tinkling. Laboratory studies show a leukocyte count of 11,500/mm3 and a serum albumin level of 2.5 mg/dL. Acute abdominal series shows dilated loops of small bowel; no other abnormalities are noted. Which of the following is the most likely diagnosis? a) Intussusception b) Crohn's c) UC d) Enteritis 2/2 radiation therapy e) Gallstone ileus

d) Enteritis 2/2 radiation therapy The patient's clinical history (radiation) puts her at life time risk for secondary enteritis. While the patient may have an early small bowel obstruction, enteritis secondary to radiation is the most likely answer. Crohn's and ulcerative colitis are unlikely to present at this age or in this clinical picture.

A 38 year old man developed severe, right lower quadrant abdominal pain 6 hours prior to his emergency department presentation. Prior to this, he was feeling well. He has poor appetite, nausea, vomiting, diarrhea, and reports a subjective fever at home, though he did not check his temperature. He denies dysuria, melena, hematochezia, or a history of similar symptoms. Yesterday, he reports he felt perfectly well. He has no significant past medical, surgical or family history. He takes no medications or supplements daily and has no known drug allergies. On physical examination, which of the following would most support your clinical diagnosis: a) Observing that the patient lies with his legs extended and resists flexion b) Normal bowel sounds c) Right upper quadrant tenderness, worse with inspiration d) Reproduction of his right lower quadrant abdominal pain with internal rotation of his flexed right hip e) Feeling Hungry

d) Flexion and internal rotation of the right hip will put the obturator internus muscle on stretch. If an inflamed appendix is in contact with this muscle, right lower quadrant abdominal pain will be reproduced. This is obturator sign and would support the dx of appendicitis. Palpation in the Right upper quadrant that exhibits tenderness is typically associated with gallbladder disease. Patients with inflammation of the peritoneum tend to prefer their legs flexed to reduce stretch on the peritoneum. This position also relaxes the psoas muscle which may be secondarily inflamed from appendicitis. Normal bowel sounds may or may not occur in a patient with appendicitis. The presence of obturator sign would be much more helpful to support your clinical diagnosis.

A 38 year-old female presents with frequent bouts of headaches. She has been on three different anti-hypertensive medications since the age of 36. Despite maximizing the dosage of the anti-hypertensive medications, her systolic blood pressure continues to remain in the 160's. She is currently taking an ACE-Inhibitor, a B-Blocker, and a Calcium channel blocker. She had CT scan when she was 30 years old after an automobile accident and was told that she may have small nodular mass on her right adrenal gland. On physical exam, she is well appearing with no palpable mass on abdominal exam. Her temperature is 98.7, Blood pressure is 160/60, HR of 80. Her fundoscopic examination is unremarkable. Routine blood tests showed the following: BMP Sodium: 144 meq/L Potasium: 2.9 meq/L Chloride: 103 meq/L Bicarbonate: 30 meq/L Creatinine: 1.1mg/dL Urinalysis: Normal Urine Sodium: low Urine Potassium: high What is the next best step in management for this patient? a) Dexamethasone suppression b) Renal Angiography c) CT scan of the abdomen and pelvis d) Plasma renin/aldosterone level e) Measurement of plasma metanephrine and normetanephrine levels

d) Plasma renin/aldosterone level This patient has an aldosteronoma. The differential diagnosis for patient who presents with uncontrolled hypertension includes Pheochromocytoma, Aldosteronoma, Cushings Disease, and Renal Hypertension. The patient above presented with uncontrolled hypertension accompanied by low serum potassium level and slightly elevated serum sodium level. Furthermore, this patient also has elevated urine potassium which is consistent with the diagnosis of aldosteronoma.

Eight weeks after a 44-year-old man underwent laparoscopic adjustable gastric band placement, he comes to the emergency department because of a 4-day history of worsening nausea and pain after eating. He has been unable to tolerate any solid or liquid food for the last 2 days. On physical examination, pulse rate is 110/min and decreases to 85/min after administration of 2 L of intravenous crystalloid infusion. His vital signs are otherwise normal. The abdomen is soft and nontender to palpation. Laboratory studies show no abnormalities. Which of the following is the most appropriate next step in management? a) Upper endoscopy with possible stent placement b) Emergent laparoscopy c) Mesenteric angiography d) Upper gastrointestinal contrast study e) Nasogastric tube decompression and serial abdominal examinations

d) Upper gastrointestinal contrast study Inability to tolerate a diet after a laparoscopic band suggests either the band is too tight or has slipped. An UGI series or even plain X Rays can diagnose a slipped band but, if the band is too tight then an UGI series will be needed. Emergent treatment is to decompress the band by removing fluid from the port (band adjustment).

A 58 year old man is recently diagnosed with an abdominal aneurysm measuring 3.8 centimeters by duplex ultrasound. He smokes one pack per day of cigarettes and has hypercholerstolemia and GERD. He is put into an aneurysm surveillance program. What are the best recommendations for medical management of patients with small aneurysms? a) smoking cessation b) statin therapy c) walking regimen d) all of the above

d) all of the above Smoking has been linked to an increased aneurysm expansion rate, and thus, smoking cessation is the most important modifiable risk factor that can be changed by patients. Several studies also demonstrate that statin use is associated with a decreased aneurysm expansion rate. Finally, lack of exercise and stagnant flow from a prolonged sedentary condition is associated with aneurysm formation, and exercise may be beneficial in prevention of aneurysm growth. In addition, exercise helps maintain cardiac health which is necessary for optimal outcomes if surgery is ever needed.

Rovsing's sign is said to be positive when the patient feels pain in the right lower quadrant with which of the following maneuvers? a) active flexion of R hip b) passive stretch of R hip c) passive extension of R hip d) deep palpation in LLQ e) internal rotation of R hip

d) deep palpation in LLQ Rovsing's sign is elicited when pressure applied in the left lower quadrant produces pain to the right lower quadrant. The psoas sign is elicited by stretch or active flexion of the right hip; the patient with acute appendicitis and a retrocecal appendix will typically have pain with these maneuvers as the inflamed organ lies upon the right iliopsoas muscle. The obturator sign is elicited with the patient in the supine position, with passive rotation of the flexed right thigh; pain with this maneuver suggests a pelvic location of the acutely inflamed appendix. *All of these are considered peritoneal signs and should be sought in the examination of a patient with acute appendicitis. Note that, depending upon the anatomic location of the appendix, not all signs may be present.*

appendicitis is thought to be caused by a) idiopathic inflammation b) mucocele tumor c) bacterial infection d) obstruction of appendiceal lumen e) subclincial trauma

d) obstruction of appendiceal lumen The pathophysiology of appendicitis is not uniform across ages. Thus, while appendicitis can occur (much less commonly) due to ischemic, infectious, or oncologic process, the overall prevailing mechanism is lumenal obstruction that causes proximal appendiceal inflammation and subsequent rupture.

A previously healthy 25 year old man presents to the emergency department with progressive right lower quadrant abdominal pain over the last 24 hours. He reports no recent weight loss and no blood in his stool. His vital signs in the emergency department show: Temperature: 38.5C HR 110 RR 24 BP 110/70 SaO2 100% on room air On physical examination he has severe tenderness to palpation in his right lower quadrant, without significant tenderness in the remainder of his abdomen. Labs are remarkable for a WBC of 18,000 with 15% bandemia A CT scan of his abdomen and pelvis shows a thickened appendix dilated to 1.5 cm with surrounding fat stranding. Which of the following is the most likely etiology for his symptoms? a) lymphoma involving base of appendix b) periappendiceal abscess c) Crohn's d) obstruction of appendiceal lumen by a piece of stool e) cecal cancer that includes the appendiceal orifice

d) obstruction of appendiceal lumen by a piece of stool This patient has acute appendicitis. The most likely etiology for this is obstruction of the appendiceal lumen by a fecalith or lymphoid hyperplasia. Lymphoma involving the base of the appendix would be an unusual etiology for appendicitis, and additional lymphadenopathy would be expected on physical examination or CT. Crohn's disease can mimick appendicitis but the CT would more typically show inflammatory changes involving the terminal ileum or a more extensive portion of the cecum. A peri-appendiceal abscess was not seen on the CT.

A 56-year-old man with type 2 diabetes mellitus underwent an exploratory laparotomy with lysis of adhesions and resection for a small bowel obstruction 4 days ago. His only medications are insulin and oral analgesics. On physical examination, the temperature is 103.3°F (39.6°C), pulse is 110/min and regular with no murmur, respirations are 18/min, and blood pressure is 110/70 mm Hg. Breath sounds are decreased bilaterally. The abdomen is soft; there is tenderness at the middle of the incision, which is hyperemic. Which of the following is the most appropriate next step in management? a) begin broad-spectrum abx b) order vigorous resp therapy c) CT abd d) open incision e) continue to observe

d) open incision The patient is presenting with signs and symptoms of an infection. The examining physician needs to consider multiple sources that may harbor an infection on post operative day four. The wound, specially in a clean contaminated surgery such as a bowel resection, is one of these locations. With tenderness and hyperemia around the wound, opening the wound becomes necessary.

The most common surgical complication associated with appendectomy is which of the following? a) hemorrhage b) unsightly scarring c) inadvertent injury to the cecum d) wound infection e) recurrent appendicitis

d) wound infection Wound infection is common, usually confined to the subcutaneous tissues, and responds following opening of the wound. Dehiscence is uncommon with a McBurney incision. Iatrogenic cecal injury can occur with either open or laparoscopic approach but is uncommon, as is hemorrhage. Recurrent appendicitis is not possible; further inflammation or fistula should raise concern for Crohn's disease. Scarring is usually minimal due to the small size of the surgical wounds, even if left open to heal by secondary intent in the face of gross perforation.

include in RLQ ddx

diverticulitis, Crohn's disease, pyelonephritis, gastroenteritis, PID, ureteral stone Gastroenteritis can be associated with abdominal pain, nausea and vomiting, but diarrhea is often a prominent symptom. Crohn's disease, involving the terminal ileum, can mimic appendicitis and may be diagnosed intraoperatively. If the appendix is not inflamed, appendectomy is recommended to prevent future confusion. A right ureteral stone can have symptoms similar to a retrocecal appendicitis. Diverticulitis of the right colon or a redundant sigmoid colon can cause symptoms in the right lower quadrant, typically in the older patient. Gynecological pathology, such as pelvic inflammatory disease, can mimic appendicitis in the female of reproductive years. A history of vaginal discharge and cervical motion tenderness are two distinguishing features.

A 54 year old man is diagnosed with a right popliteal artery aneurysm measuring 3.5 centimeters. What are the chances he has a coexisting abdominal aortic aneurysm? a) 90% b) 10% c) 75% d) 3.5% e) 50%

e) 50% In a patient with popliteal artery aneurysms, there is approximately 50% risk of coexisting abdominal aortic aneurysm. Conversely, if a patient has an abdominal aortic aneurysm, there is approximately 3.5% risk of a popliteal artery aneurysm.

You are discharging a 35 year old, otherwise healthy man following a laparoscopic appendectomy for an uncomplicated acute appendicitis. The patient has been ambulating and tolerated breakfast. On examination the wounds appear to be clean, without drainage. Vital signs include a blood pressure of 120/75 mmHg, pulse of 75 per minute, respirations of 12 per minute. Which of the following is the most appropriate advice for this patient on discharge? a) come to the Emergency Department if you experience mild pain or constipation b) do not shower or bathe for three days c) redness and drainage from the incisions is normal d) continue with a liquid diet until the 1st post op visit e) a low grade fever is normal in the first 24-48 hrs after surgery

e) A low grade fever is normal in the first 24-48 hrs after surgery A low grade (<38.0° C) fever is not uncommon after any surgical procedure due to the triggering of the stress response system. A higher fever or chills should be concerning. Patients should be alerted to potential complications such as wound infection characterized by erythema, increasing pain or purulent drainage. Constipation can be common while patients are taking narcotic pain medication. Showers are permitted beginning 24 hours after the procedure, once the skin has sealed. Submerging the incision underwater in a tub or pool is usually discouraged until sutures/staples are removed at the post-operative office visit.

A previously healthy 42 year-old woman presents with a 12-hour history of increasing abdominal pain, most marked in the right lower quadrant. On examination the abdomen is diffusely tender with localized tenderness and rebound in the right lower quadrant. Vital signs include a blood pressure of 135/83 mmHg, a pulse of 102 per minute, respirations of 18 per minute, and a temperature of 38.2° C. Laboratory studies include a hemoglobin of 12.3; the WBC is 12,500, with a left shift. Which of the following is the most appropriate imaging study based on the patient's history? a) MRI b) Flat and upright films of chest and abdomen c) CT of abdomen and pelvis with contrast d) Technetium sulfur-colloid scan e) Abdominal u/s f) No imaging studies needed

e) Abdominal u/s Imaging studies should be considered only when they will aid in narrowing the differential diagnosis and thus change the patient's management. While appendicitis remains high on the differential in this patient, a 12 hour history of increasing and diffuse abdominal pain in a female (i.e. relatively early to consider perforation) places ovarian cyst/rupture higher on the list. This patient requires a detailed gynecological examination. The best imaging, in this case, would be a pelvic ultrasound examination.

A 45-year-old man is brought to the Emergency Department by his wife because of abdominal pain and fever. He underwent laparoscopic appendectomy four days ago for a perforated appendicitis. He appears relatively healthy and has eaten breakfast; his body habitus is slightly obese. Vital signs include a blood pressure of 135/83 mmHg, a pulse of 102 per minute, respirations of 18 per minute, and a temperature of 38.2°C. On physical examination there are well healing port sites. The abdomen appears non-distended. There is mild diffuse abdominal discomfort to palpation. Bowel sounds are absent. The hemoglobin is 13.5. The WBC is 15,000, with an increase in the white blood cell count. What is the most appropriate next step in management? a) diagnostic laparoscopy b) oral abx c) abdominal u/s d) outpatient f/u within 2 days e) CT-scan

e) CT-scan With an elevated WBC and fever accompanying abdominal discomfort on post-op day #4, there should be strong suspicion for a complication such as an abscess which requires immediate evaluation. There is no evidence of local wound infection, therefore antibiotics alone are insufficient. If an intra-abdominal abscess exists, it can be managed with percutaneous drainage, rather than a return to the OR. CT scan has a much higher sensitivity for detecting abscess than an ultrasound, particularly since there may be an element of ileus that can negatively impact the quality of the ultrasound.

A 63-year-old woman comes to the emergency department because of a 2-day history of increasing abdominal distention and obstipation. She has vomited twice in the past 8 hours. She has a 2-month history of diffuse, dull lower abdominal pain. She has noted a recent change in bowel habits, characterized by passing hard, pellet-like stools alternating with loose stools. She has lost 5 lb (2.2 kg) over the past 2 months. She has osteoarthritis and frequent episodes of heartburn. Her current medications are ibuprofen and omeprazole. On physical examination, her temperature is 98.6°F (37.0°C), pulse is 90/min and regular, respirations are 18/min, and blood pressure is 118/80 mm Hg. No murmurs are heard. The lungs are clear to auscultation and percussion. Abdominal examination shows distention with high-pitched, tinkling bowel sounds and diffuse, mild tenderness. Which of the following is the most likely diagnosis? a) SBO b) Colonic Crohn's c) Sigmoid volvulus d) Fecal impaction e) Colon CA

e) Colon CA Of the five options, colon cancer is the most likely. The clinical picture (specifically the physical examination) is not consistent with small bowel obstruction, impaction, or volvulus. The pt's age and presentation are not consistent with crohn's.

Three months following drainage of a perianal abscess, a 33-year-old man comes to see the physician because of intermittent perianal pain and drainage. He notices an occasional drop of blood. The drainage does not seem related to the timing of defecation. His postoperative course had been uneventful until approximately 3 weeks ago. On physical examination, his temperature is 98.6°F (37.0°C), pulse is 70/min and regular, respirations are 14/min, and blood pressure is 120/70 mm Hg. Which of the following will most likely be discovered on rectal examination? a) Recurrent abscess b) Internal hemorrhoids c) External hemorrhoids d) Anal fissure e) Fistula-in-ano

e) Fistula-in-ano This man has a fistula-in-ano or anal fistula, a fistula between the anorectal junction and the skin. These fistula usually arise after drainage of a perianal or perirectal abscess and represent a tube of chronic granulation tissues that fails to heal after drainage of the abscess. Perianal and perirectal abscesses usually start in the anal crypts and erode through the sphincter muscle. A perianal abscess occurs if the infection erodes to the perianal skin through the subcutaneous portion of the sphincter. A perirectal abscess occurs when the abscess erodes through the deeper portions of the sphincter into the perirectal and ischiorectal spaces. Sometimes the abscesses drain spontaneously but often require incision and drainage. Any patient with a perianal or perirectal abscess should be advised that a fistula is very likely after drainage, and this can occur even months later after presumed healing. The opening on the perianal or perirectal skin is the secondary opening. The primary opening is at the dentate line where the initiating cryptoglandular abscess occurred. A recurrent abscess usually occurs for the same reason as a fistula, however, the secondary opening (skin) has healed and spontaneous drainage has not occurred. Internal hemorrhoids consist of submucosal tissue with dilated hemorrhoids and overlying rectal mucosa. There are three main columns of hemorrhoids - right anterior, right lateral and left posterior. One or all three of the columns may be enlarged and prolapse. The hemorrhoids are occasionally associated with painless discharge of mucous and intermittent bleed often with defecation. External hemorrhoids are covered with squamous mucosa and are located under the anoderm in the anal canal. Anoderm has somatic innervation, and thrombosis and inflammation of external hemorhhoids causes moderate to severe pain. Bleeding from thrombosed hemorrhoids occurs when the clot erodes through the skin. Minor anal trauma from constipation (usual history) or diarrhea can lead to an acute anal fissure. The patient may not have pain initially but may see bright red blood on the toilet tissue. Pain may occur with repeated episode. Initially the pain is with defecation. With time the pain may be initiated by defecation but lingers long afterward due to sphincter spasm which leads to ischemia of the anoderm which in turns leads to a chronic anal fissure which is manifested as a small ulcer and scarring into the internal anal sphincter. Patients avoid defecating due to the pain leading to more constipation and propagation of the problem. Chronic anal fissures are usually located in the posterior midline, the least distensible area of the sphincter mechanism. Pain and sphincter spasm makes it very difficult to do a digital rectal exam without anesthesia. There is often a prominent skin tag or sentinel pile that is distal to the fissure that lies in the anal canal. This may appear edematous and be painful when palpated, and is often confused as a thrombosed hemorrhoid or anal warts (condyloma accuminata) by inexperienced clinicians. Carefully spreading the anal skin and sphincter will reveal the small ulcer proximal to the tag, but this can be difficult to accomplish without anesthesia in many patients.

A 60-year-old man is evaluated for a 5-day history of lower abdominal pain and tenesmus. For the past 2 days, the abdominal pain has increased, and he has had nausea without vomiting. He has 2 previous episodes of similar symptoms that were treated on an outpatient basis. The patient has a history of chronic constipation that he manages with magnesium hydroxide. On physical examination, his temperature is 100.8°F (38.2°C), pulse is 90/min and regular, respirations are 16/min, and blood pressure is 140/90 mm Hg. No murmurs are heard. The lungs are clear to auscultation and percussion. There is significant tenderness in the left lower quadrant with guarding but no rebound. Laboratory studies show a leukocyte count of 12,400/mm3 and hemoglobin of 12 g/dL. Which of the following is the most likely diagnosis? a) ischemic colitis b) crohn's c) sigmoid volvulus d) carcinoma of the colon e) diverticulitis

e) diverticulitis Diverticulitis often presents with low grade temps, left lower quadrant pain and tenderness, and leukocytosis. The history of previous similar symptoms treated as an outpatient is also consistent with the diagnosis of diverticulitis.

A 62 year old male underwent a CT scan for evaluation for right lower quadrant pain to rule out appendicitis. His medical history is significant for hypertension, hypercholesterolemia, and recent MI three months ago that required cardiac catheterization and placement of drug eluting stent. His CT scan was unremarkable for appendicitis but incidentally he was found to have 2.5 cm adrenal nodule on the right side. Further work up revealed normal serum and urine catecholamine levels, and a plasma aldosterone/renin ratio of 31. His current medication includes a beta-blocker, ACE-inhibitor, and calcium channel blocker. On physical exam, he was normotensive at 120/80, P: 65 and otherwise in good health. What would you advise this patient at this point? a) Open adrenalectomy for functional mass b) Observe the mass as it is a small incidentaloma that is hormonally inactive and the there is no benefit to excision c) Laparoscopic adrenalectomy for functional adrenal mass d) Laparoscopic adrenalectomy for non-functional adrenal mass e) Observe the mass as the risk outweighs the benefit at this point

e) Observe the mass as the risk outweighs the benefit at this point This is a functional aldosteronoma with plasma aldosterone/renin ratio over 30. In a normal circumstance, where there is a proven functional adrenal nodule, the patient would undergo adrelenalectomy. In this patient with recent MI with drug eluting stent, he will need to be on anticoagulants for more than three months. Furthermore, his blood pressure is currently is well controlled with his anti-hypertensive medications. The risk of this patient undergoing the operation outweighs the benefit at this point. He will be better served with observation until he is medically optimized or he becomes more symptomatic.

A 40-year-old man comes to the emergency department because of a 10-day history of perianal pain with discharge. His medical history is unremarkable, and he takes no medications. On physical examination, his temperature is 99.5°F (37.5°C), pulse is 90/min and regular, respirations are 14/min, and blood pressure is 130/80 mm Hg. Examination shows a 2-cm hole from the anal verge in the left posterior lateral position; the opening is draining a small amount of pus. The surrounding tissue is mildly tender. Perioperative examination of the anal canal is most likely to reveal an internal opening to a fistula at which of the following locations? a) Left posterior rectum b) Anterior mid dentate line c) Left posterior dentate line d) Left anterior rectum e) Posterior mid dentate line

e) Posterior mid dentate line This man has a fistula- in- ano or anal fistula, a fistula between the anorectal junction and the skin. These fistula usually arise after drainage of a perianal or perirectal abscess and represent a tube of chronic granulation tissues that fails to heal after drainage of the abscess. Perirectal abscesses usually start in the anal crypts and erode through the sphincter muscle. A perianal abscess occurs if the infection erodes to the perianal skin through the subcutaneous portion of the sphincter. A perirectal abscess occurs when the abscess erodes through the deeper portions of the sphincter into the perirectal and ischiorectal spaces. Sometimes the abscesses drain spontaneously but often require incision and drainage. Any patient with a perianal or perirectal abscess should be advised that a fistula is very likely after drainage, and this can occur even months later after presumed healing. The opening on the perianal or perirectal skin is the secondary opening. The primary opening is at the dentate line where the initiating cryptoglandular abscess occurred. The path that the fistula takes is predictable according to Goodsall's rule. When the secondary opening (external) is anterior to the mid- transverse plane of the anus, the fistula follows a radial course to the anus and the primary opening at the dentate line. If the secondary opening is posterior to the transverse plane, the fistula will follow a course back to the posterior midline dentate line. The mechanism for this is not clear but may be related to failure of fusion of the longitudinal muscle and the external sphincter in the posterior midline. For secondary openings more than 2 cm from the anal verge, Goodsall's rule may not apply as even anterior openings may track back to the posterior midline anus. Goodsall's rule does not apply to patients with Crohn's disease.

An 18 year old man presents to the Emergency Department with a 14 hour history of abdominal pain which has now localized to the right lower quadrant. On physical examination, there is tenderness of deep palpation in the right lower quadrant, without guarding or rebound. The Rovsing's sign is negative. Which of the following additional physical findings would be most consistent with a diagnosis of appendicitis if positive? a) Romberg's sign b) Murphy's sign c) Carnett's sign d) Chvostek's sign e) Psoas sign

e) Psoas sign Right lower quadrant pain with passive (or active) extension of the right lower extremity. This typically indicates a process that is irritating the right psoas muscle. (Note: the patient is on their side during this examination)

A 45-year-old woman with a body mass index of 48 kg/m2 is scheduled to undergo Roux-en-Y gastric bypass surgery. She has sleep apnea and a 6-year history of type 2 diabetes mellitus that is currently managed with metformin. On physical examination, temperature is 98.4°F (36.9°C), pulse is 88/min, respirations are 18/min, and blood pressure is 130/90 mm Hg. The remainder of the physical examination is unremarkable. This patient is most prone to which of the following vitamin deficiencies after surgery? a) Folic acid b) Vitamin E c) Vitamin K d) Vitamin A e) Vitamin B12

e) Vitamin B12 In a gastric bypass, the patient may be deficient in any of these vitamins but is most prone to B12 deficiency due to the inability of the food intake to mix with intrinsic factor thus reducing intestinal absorption of B12.

An 18 year old girl presents with a 12 hour history of lower abdominal pain, which has localized to the right lower quadrant. She has not had anything to eat or drink for the last 12 hours. Her last menstrual period was 2 weeks ago. She is not sexually active. On physical examination the abdomen is tender in the lower quadrants, with the greatest tenderness to deep palpation in the right lower quadrant. Pelvic examination demonstrates no cervical motion tenderness. No masses are appreciated. Vital signs include a blood pressure of 120/80 mmHg, a pulse of 110 per minute, respirations of 18 per minute, and a temperature of 38.0° C. Which of the following laboratory studies would be most typically associated with this history? a) 3+ glucose on UA b) Na of 129 c) 30-40 RBC per HPF on UA d) BUN of 40 e) WBC 13.1 f) Hematocrit of 25

e) WBC 13.1 Lab results need to be interpreted in light of the patient's clinical picture. While later/complicated appendicitis can have a wide spectrum of presentations, early appendicitis is typically seen with a mildly elevated WBC. While a few RBCs can be seen in the urine in the presentation of appendicitis, specially if the appendix is in the pelvis, these tend to be less then 5 cells per HPF. An elevated glucose in the urine or an elevated BUN are not typically seen in early appendicitis. Hyponatremia is not seen in early appendicitis either.

Which of the following is the most useful landmark to find the appendix on CT scan? a) terminal ileum b) R ureter c) R psoas muscle d) R ASIS e) cecum

e) cecum While the appendix is anatomically related to all structures listed, this relationship can be quiet variable. The relationship of the appendix to the cecum is most consistent due to their anatomical continuity.

Which of the following laboratory results would be most typical for mesenteric adenitis? a) elevated hematocrit b) elevated platelet count c) elevated neutrophil count d) elevated eosinophil count e) elevated lymphocyte count

e) elevated lymphocyte count Mesenteric adenitis by definition is enlargement of the mesenteric lymph nodes and therefore is associated with an increase in the relative proportion of lymphocytes.

A 72-year-old woman comes to the emergency department with a 2-day history of vomiting and intermittent colicky abdominal pain. She has not had any recent weight loss. For the past 10 years, she has had postprandial upper abdominal pain that resolves with self medication with an analgesic. She has no other significant medical history, and she takes no other medications. On physical examination, her temperature is 96.8°F (36.0°C), pulse is 90/min and regular, respirations are 22/min, and blood pressure is 130/90 mm Hg. No murmurs are heard. The lungs are clear to auscultation and percussion. The abdomen is distended and mildly tender to deep palpation. Bowel sounds are high pitched and tinkling. A plain radiograph of the abdomen shows branching air collections projecting over the liver with distended small bowel and decompressed colon. Which of the following is the most likely diagnosis? a) adhesive SBO b) Internal hernia c) lymphoma d) diverticulitis e) gallstone ileus

e) gallstone ileus The clinical picture and distended small bowel loops with a decompressed colon are characteristic of a small bowel obstruction. Adhesive disease is unlikely with the lack of past surgical intervention. The presence of air in branching pattern over the liver is characteristic of biliary air seen in patient presenting with small bowel obstruction due to a gallstone that is typically stuck at the ileo-cecal valve.

A 25 year old woman is seen in the clinic three weeks following an appendectomy for acute appendicitis. The wound is well - healed. The pathology report identifies a 1.2 cm well-differentiated carcinoid tumor at the tip of the appendix, as well as acute appendicitis. What would be the next most appropriate step in diagnosis? a) U/s b) PET c) CT d) MRI e) no further diagnostic studies

e) no further diagnostic studies Well-differentiated carcinoid tumors, less than 2cm in diameter, that are incidentally found with a clear margin do not require additional diagnostic studies or workup.

Based on the pathology of a well-differentiated carcinoid tumor on the appendix, what is the most appropriate therapeutic action? a) chemo b) retroperitoneal lymph node dissection c) right hemicolectomy d) radiation therapy e) observation

e) observation Well-differentiated carcinoid tumors, less than 2cm in diameter, that are incidentally found with a clear margin do not require any further intervention. For larger carcinoids, or those with more aggressive growth, further diagnostic studies and/or therapeutic interventions maybe required.

Which of the following symptoms is most indicative of acute appendicitis in a patient with right lower quadrant pain? a) rigors and high fever b) prior episodes of abdominal pain c) anorexia d) bloody diarrhea e) pain preceding vomiting

e) pain preceding vomiting Appendicitis starts with obstruction of the appendix which causes pain, and progresses to ileus which leads to the symptoms of nausea and vomiting. Rigors and high fever would be signs of advanced appendicitis due to perforation and peritonitis. Prior episodes of abdominal pain are rare with appendicitis. Anorexia is almost always associated with appendicitis, but is not specific. Bloody stools are not seen with appendicitis but can be seen with Meckel's diverticulum due to heterotopic gastric mucosa and acid production leading to ulceration.

A 23-year-old woman presents to the emergency department with a 2-day history of cramping abdominal pain, nausea, and vomiting. She also reports passing dark stools. Upon questioning, she recalls having similar episodes during the past two years, each of which resolved spontaneously without treatment. She has no history of abdominal operations. She takes no medications. On physical examination, her temperature is 98.4°F (36.9°C), pulse is 122/min, respirations are 22/min, and blood pressure is 132/69 mm Hg. She has abdominal distention and diffuse tenderness without rebound or guarding. A stool specimen is positive for occult blood. Contrast-enhanced computed tomography is most likely to show inflammation of which portion of the intestine? a) distal third of descending colon b) mid-jejunum c) transverse colon d) stomach e) terminal ileum

e) terminal ileum The patient's presentation, physical exam, and age are most consistent with a complication of Crohn's disease. Evaluation with a CT scan would be appropriate at this time and would probably show thickening and inflammation at the terminal ileum (most common location in the GI tract to be affected by Crohn's disease).

A 32 year-old woman undergoes an uncomplicated laparoscopic appendectomy for acute suppurative appendicitis. At the time of office followup, one week after surgery, what findings would be expected and which would be unexpected?

expected: incisional pain that is decreasing with time, pruritis of the surgical wound not expected: frequent urination/urgency, wound erythema, incisional drainage, intermittent fevers, obstipation, persistent nausea One week following surgery, a patient should have appropriate incisional pain and tenderness that are improving daily. Many patients will note pruritis as a healthy wound heals; in the absence of a pathologic cause (i.e., contact dermatitis secondary to surgical tape) they should be reassured. Increasing erythema of the wound or drainage from the incision, however, are hallmarks of a wound infection. Likewise, the patient should no longer have significant nausea, or any fevers. These complaints should prompt a search for a specific cause, intra-abdominal abscess for example. Urinary frequency or urgency may be a sign of a urinary tract infection (presumably the patient had a Foley catheter at the time of operation), or more ominously can also be associated with a pelvic abscess.

A 65-year old woman with a history of mild hypertension, presents with a one week history of fever, anorexia, abdominal pain and evidence of dehydration. Vital signs include a blood pressure of 140/85 mmHg, a pulse of 90 per minute, respirations of 22 per minute, and a temperature of 101.2° F. On examination her abdomen is soft. There is localized mild tenderness and voluntary guarding in the right lower quadrant with associated fullness and a tender palpable mass. The most likely diagnosis is? a) cecal diverticulitis b) Meckel's diverticulitis c) pelvic abscess d) tubo-ovarian abscess e) sigmoid diverticulitis f) periappendiceal abscess

f) periappendiceal abscess The patient's clinical picture is most consistent with an intraabdominal infectious process. The differential diagnosis in a 65 year old women includes, most commonly, complicated appendicitis and diverticulitis (sigmoid more likely than cecal). Tubo-ovarian abscess and Meckel's being less likely in this age group. While complicated diverticulitis remains in the differential diagnosis, the right lower quadrant location and the relative statistical frequency of appendicitis make it the most likely diagnosis.

A 35 year old woman presents with abdominal pain and suspected appendicitis. A FOCUSED history should include a review of which categories?

include: respiratory, gynecologic, trauma, GI, urologic, MSK exclude: CV, neuro, EENT A review of systems should include the gastrointestinal system as well as neighboring urological and gynecologic organs as symptoms can be very similar, such as pelvic inflammatory disease and cystitis. Symptoms related to these disease processes should be actively elicited in the history. History of trauma to the abdomen is pertinent as well as any musculoskeletal disorder that may cause abdominal wall pain. Absence of symptoms associated with an upper respiratory infection should be sought to exclude the possibility of mesenteric adenitis.

U/s is sensitive for diagnosing

yes: appendicitis, PID, ectopic pregnancy, pelvic fluid, ovarian torsion no: pneumoperitoneum, Crohn's, ureteropelvic junction stones, Meckel's diverticulum, enteric fistula, cystitis While ultrasound has the disadvantage of being operator dependent and patient depended (less helpful in obese patients or patients with exquisite tenderness), it remains an important tool in the evaluation of abdominal and pelvic pain. In the correct setting, it has been shown to be helpful in the evaluation of appendicitis, ectopic pregnancy, ovarian torsion, pelvic fluid, and/or pelvic inflammatory disease.


Set pelajaran terkait

Cultural Anthropology - Final Exam

View Set